Programs & Examples On #Ltrace

Add JVM options in Tomcat

If you start tomcat from startup.bat, you need to add a system variable :JAVA_OPTS as name and the parameters that you wants (in your case :

-agentpath:C:\calltracer\jvmti\calltracer5.dll=traceFile-C:\calltracer\call.trace,filterFile-C:\calltracer\filters.txt,outputType-xml,usage-uncontrolled -Djava.library.path=C:\calltracer\jvmti -Dcalltracerlib=calltracer5

How to create a notification with NotificationCompat.Builder?

simple way for make notifications

 NotificationCompat.Builder builder = (NotificationCompat.Builder) new NotificationCompat.Builder(this)
            .setSmallIcon(R.mipmap.ic_launcher) //icon
            .setContentTitle("Test") //tittle
            .setAutoCancel(true)//swipe for delete
            .setContentText("Hello Hello"); //content
    NotificationManager notificationManager = (NotificationManager) getSystemService(Context.NOTIFICATION_SERVICE);

    notificationManager.notify(1, builder.build()
    );

How to mock location on device?

Make use of the very convenient and free interactive location simulator for Android phones and tablets (named CATLES). It mocks the GPS-location on a system-wide level (even within the Google Maps or Facebook apps) and it works on physical as well as virtual devices:

Website: http://ubicom.snet.tu-berlin.de/catles/index.html

Video: https://www.youtube.com/watch?v=0WSwH5gK7yg

How should I log while using multiprocessing in Python?

Yet another alternative might be the various non-file-based logging handlers in the logging package:

  • SocketHandler
  • DatagramHandler
  • SyslogHandler

(and others)

This way, you could easily have a logging daemon somewhere that you could write to safely and would handle the results correctly. (E.g., a simple socket server that just unpickles the message and emits it to its own rotating file handler.)

The SyslogHandler would take care of this for you, too. Of course, you could use your own instance of syslog, not the system one.

What is the difference between Collection and List in Java?

Collection is a high-level interface describing Java objects that can contain collections of other objects. It's not very specific about how they are accessed, whether multiple copies of the same object can exist in the same collection, or whether the order is important. List is specifically an ordered collection of objects. If you put objects into a List in a particular order, they will stay in that order.

And deciding where to use these two interfaces is much less important than deciding what the concrete implementation you use is. This will have implications for the time and space performance of your program. For example, if you want a list, you could use an ArrayList or a LinkedList, each of which is going to have implications for the application. For other collection types (e.g. Sets), similar considerations apply.

JavaScript: Collision detection

The first thing to have is the actual function that will detect whether you have a collision between the ball and the object.

For the sake of performance it will be great to implement some crude collision detecting technique, e.g., bounding rectangles, and a more accurate one if needed in case you have collision detected, so that your function will run a little bit quicker but using exactly the same loop.

Another option that can help to increase performance is to do some pre-processing with the objects you have. For example you can break the whole area into cells like a generic table and store the appropriate object that are contained within the particular cells. Therefore to detect the collision you are detecting the cells occupied by the ball, get the objects from those cells and use your collision-detecting function.

To speed it up even more you can implement 2d-tree, quadtree or R-tree.

Error using eclipse for Android - No resource found that matches the given name

I renamed the file in res/values "strings.xml" to "string.xml" (no 's' on the end), cleaned and rebuilt without error.

Difference between 'cls' and 'self' in Python classes?

cls implies that method belongs to the class while self implies that the method is related to instance of the class,therefore member with cls is accessed by class name where as the one with self is accessed by instance of the class...it is the same concept as static member and non-static members in java if you are from java background.

How do I exit from a function?

I'd suggest trying to avoid using return/exit if you don't have to. Some people will devoutly tell you to NEVER do it, but sometimes it just makes sense. However if you can structure you checks so that you don't have to enter into them, I think it makes it easier for people to follow your code later.

How do I activate C++ 11 in CMake?

The CMake command target_compile_features() is used to specify the required C++ feature cxx_range_for. CMake will then induce the C++ standard to be used.

cmake_minimum_required(VERSION 3.1.0 FATAL_ERROR)
project(foobar CXX)
add_executable(foobar main.cc)
target_compile_features(foobar PRIVATE cxx_range_for)

There is no need to use add_definitions(-std=c++11) or to modify the CMake variable CMAKE_CXX_FLAGS, because CMake will make sure the C++ compiler is invoked with the appropriate command line flags.

Maybe your C++ program uses other C++ features than cxx_range_for. The CMake global property CMAKE_CXX_KNOWN_FEATURES lists the C++ features you can choose from.

Instead of using target_compile_features() you can also specify the C++ standard explicitly by setting the CMake properties CXX_STANDARD and CXX_STANDARD_REQUIRED for your CMake target.

See also my more detailed answer.

:: (double colon) operator in Java 8

Usually, one would call the reduce method using Math.max(int, int) as follows:

reduce(new IntBinaryOperator() {
    int applyAsInt(int left, int right) {
        return Math.max(left, right);
    }
});

That requires a lot of syntax for just calling Math.max. That's where lambda expressions come into play. Since Java 8 it is allowed to do the same thing in a much shorter way:

reduce((int left, int right) -> Math.max(left, right));

How does this work? The java compiler "detects", that you want to implement a method that accepts two ints and returns one int. This is equivalent to the formal parameters of the one and only method of interface IntBinaryOperator (the parameter of method reduce you want to call). So the compiler does the rest for you - it just assumes you want to implement IntBinaryOperator.

But as Math.max(int, int) itself fulfills the formal requirements of IntBinaryOperator, it can be used directly. Because Java 7 does not have any syntax that allows a method itself to be passed as an argument (you can only pass method results, but never method references), the :: syntax was introduced in Java 8 to reference methods:

reduce(Math::max);

Note that this will be interpreted by the compiler, not by the JVM at runtime! Although it produces different bytecodes for all three code snippets, they are semantically equal, so the last two can be considered to be short (and probably more efficient) versions of the IntBinaryOperator implementation above!

(See also Translation of Lambda Expressions)

Two decimal places using printf( )

Use: "%.2f" or variations on that.

See the POSIX spec for an authoritative specification of the printf() format strings. Note that it separates POSIX extras from the core C99 specification. There are some C++ sites which show up in a Google search, but some at least have a dubious reputation, judging from comments seen elsewhere on SO.

Since you're coding in C++, you should probably be avoiding printf() and its relatives.

Android SDK is missing, out of date, or is missing templates. Please ensure you are using SDK version 22 or later

If Android Studio directly opening your project instead of setup window, then just close the windows of all projects. Now you will able to see the startup window. If SDK is missing then it will provide option to download SDK and other required tools.

It works for me.

How to send a GET request from PHP?

Remember that if you are using a proxy you need to do a little trick in your php code:

(PROXY WITHOUT AUTENTICATION EXAMPLE)

<?php
$aContext = array(
    'http' => array(
        'proxy' => 'proxy:8080',
        'request_fulluri' => true,
    ),
);
$cxContext = stream_context_create($aContext);

$sFile = file_get_contents("http://www.google.com", False, $cxContext);

echo $sFile;
?>

Cannot delete directory with Directory.Delete(path, true)

Is it possible you have a race condition where another thread or process is adding files to the directory:

The sequence would be:

Deleter process A:

  1. Empty the directory
  2. Delete the (now empty) directory.

If someone else adds a file between 1 & 2, then maybe 2 would throw the exception listed?

JavaScript function to add X months to a date

Simple solution: 2678400000 is 31 day in milliseconds

var oneMonthFromNow = new Date((+new Date) + 2678400000);

Update:

Use this data to build our own function:

  • 2678400000 - 31 day
  • 2592000000 - 30 days
  • 2505600000 - 29 days
  • 2419200000 - 28 days

Getting rid of all the rounded corners in Twitter Bootstrap

With SASS Bootstrap - if you are compiling Bootstrap yourself - you can set all border radius (or more specific) simply to zero:

$border-radius:               0;
$border-radius-lg:            0;
$border-radius-sm:            0;

Error : Program type already present: android.support.design.widget.CoordinatorLayout$Behavior

Go to the directory where you put additional libraries and delete duplicated libraries.

removing new line character from incoming stream using sed

This might work for you:

printf "{new\nto\nlinux}" | paste -sd' '            
{new to linux}

or:

printf "{new\nto\nlinux}" | tr '\n' ' '            
{new to linux}

or:

printf "{new\nto\nlinux}" |sed -e ':a' -e '$!{' -e 'N' -e 'ba' -e '}' -e 's/\n/ /g'
{new to linux}

HTML5 Video tag not working in Safari , iPhone and iPad

I had a similar issue where videos inside a <video> tag only played on Chrome and Firefox but not Safari. Here is what I did to fix it...

A weird trick I found was to have two different references to your video, one in a <video> tag for Chrome and Firefox, and the other in an <img> tag for Safari. Fun fact, videos do actually play in an <img> tag on Safari. After this, write a simple script to hide one or the other when a certain browser is in use. So for example:

<video id="video-tag" autoplay muted loop playsinline> 
    <source src="video.mp4" type="video/mp4" />  
</video>
<img id="img-tag" src="video.mp4">

<script type="text/javascript">
    function BrowserDetection() {

    //Check if browser is Safari, if it is, hide the <video> tag, otherwise hide the <img> tag
    if (navigator.userAgent.search("Safari") >= 0 && navigator.userAgent.search("Chrome") < 0) {
        document.getElementById('video-tag').style.display= "none";
    } else {
        document.getElementById('img-tag').style.display= "none";
    }               

    //And run the script. Note that the script tag needs to be run after HTML so where you place it is important. 
    BrowserDetection();
</script>

This also helps solve the problem of a thin black frame/border on some videos on certain browsers where they are rendered incorrectly.

Python style - line continuation with strings?

Another possibility is to use the textwrap module. This also avoids the problem of "string just sitting in the middle of nowhere" as mentioned in the question.

import textwrap
mystr = """\
        Why, hello there
        wonderful stackoverfow people"""
print (textwrap.fill(textwrap.dedent(mystr)))

how to define variable in jquery

Remember jQuery is a JavaScript library, i.e. like an extension. That means you can use both jQuery and JavaScript in the same function (restrictions apply).

You declare/create variables in the same way as in Javascript: var example;

However, you can use jQuery for assigning values to variables:

var example = $("#unique_product_code").html();

Instead of pure JavaScript:

var example = document.getElementById("unique_product_code").innerHTML;

Visual Studio can't 'see' my included header files

Here's how I solved this problem.

  • Go to Project --> Show All Files.

enter image description here

  • Right click all the files in Solutions Explorer and Click on Include in Project in all the files you want to include.

enter image description here

Done :)

Select data from date range between two dates

SELECT * from Product_sales where
(From_date BETWEEN '2013-01-03'AND '2013-01-09') OR 
(To_date BETWEEN '2013-01-03' AND '2013-01-09') OR 
(From_date <= '2013-01-03' AND To_date >= '2013-01-09')

You have to cover all possibilities. From_Date or To_Date could be between your date range or the record dates could cover the whole range.

If one of From_date or To_date is between the dates, or From_date is less than start date and To_date is greater than the end date; then this row should be returned.

How to find length of a string array?

This won't work. You first have to initialize the array. So far, you only have a String[] reference, pointing to null.

When you try to read the length member, what you actually do is null.length, which results in a NullPointerException.

WorksheetFunction.CountA - not working post upgrade to Office 2010

I'm not sure exactly what your problem is, because I cannot get your code to work as written. Two things seem evident:

  1. It appears you are relying on VBA to determine variable types and modify accordingly. This can get confusing if you are not careful, because VBA may assign a variable type you did not intend. In your code, a type of Range should be assigned to myRange. Since a Range type is an object in VBA it needs to be Set, like this: Set myRange = Range("A:A")
  2. Your use of the worksheet function CountA() should be called with .WorksheetFunction

If you are not doing it already, consider using the Option Explicit option at the top of your module, and typing your variables with Dim statements, as I have done below.

The following code works for me in 2010. Hopefully it works for you too:

Dim myRange As Range
Dim NumRows As Integer

Set myRange = Range("A:A")
NumRows = Application.WorksheetFunction.CountA(myRange)

Good Luck.

How to drop a database with Mongoose?

The difficulty I've had with the other solutions is they rely on restarting your application if you want to get the indexes working again.

For my needs (i.e. being able to run a unit test the nukes all collections, then recreates them along with their indexes), I ended up implementing this solution:

This relies on the underscore.js and async.js libraries to assemble the indexes in parellel, it could be unwound if you're against that library but I leave that as an exerciser for the developer.

mongoose.connection.db.executeDbCommand( {dropDatabase:1}, function(err, result) {
  var mongoPath = mongoose.connections[0].host + ':' + mongoose.connections[0].port + '/' + mongoose.connections[0].name
  //Kill the current connection, then re-establish it
  mongoose.connection.close()
  mongoose.connect('mongodb://' + mongoPath, function(err){
    var asyncFunctions = []

    //Loop through all the known schemas, and execute an ensureIndex to make sure we're clean
    _.each(mongoose.connections[0].base.modelSchemas, function(schema, key) {
      asyncFunctions.push(function(cb){
        mongoose.model(key, schema).ensureIndexes(function(){
          return cb()
        })
      })
    })

    async.parallel(asyncFunctions, function(err) {
      console.log('Done dumping all collections and recreating indexes')
    })
  })
})

How do I extend a class with c# extension methods?

I was looking for something similar - a list of constraints on classes that provide Extension Methods. Seems tough to find a concise list so here goes:

  1. You can't have any private or protected anything - fields, methods, etc.

  2. It must be a static class, as in public static class....

  3. Only methods can be in the class, and they must all be public static.

  4. You can't have conventional static methods - ones that don't include a this argument aren't allowed.

  5. All methods must begin:

    public static ReturnType MethodName(this ClassName _this, ...)

So the first argument is always the this reference.

There is an implicit problem this creates - if you add methods that require a lock of any sort, you can't really provide it at the class level. Typically you'd provide a private instance-level lock, but it's not possible to add any private fields, leaving you with some very awkward options, like providing it as a public static on some outside class, etc. Gets dicey. Signs the C# language had kind of a bad turn in the design for these.

The workaround is to use your Extension Method class as just a Facade to a regular class, and all the static methods in your Extension class just call the real class, probably using a Singleton.

Convert Python dictionary to JSON array

One possible solution that I use is to use python3. It seems to solve many utf issues.

Sorry for the late answer, but it may help people in the future.

For example,

#!/usr/bin/env python3
import json
# your code follows

Automatically scroll down chat div

What you need to do is divide it into two divs. One with overflow set to scroll, and an inner one to hold the text so you can get it's outersize.

<div id="chatdiv">
    <div id="textdiv"/>
</div>

textdiv.html("");
$.each(chatMessages, function (i, e) {
    textdiv.append("<span>" + e + "</span><br/>");
});
chatdiv.scrollTop(textdiv.outerHeight());

You can check out a jsfiddle here: http://jsfiddle.net/xj5c3jcn/1/

Obviously you don't want to rebuild the whole text div each time, so take that with a grain of salt - just an example.

How to scroll the page when a modal dialog is longer than the screen?

Change position

position:fixed;
overflow: hidden;

to

position:absolute;
overflow:scroll;

Explain ggplot2 warning: "Removed k rows containing missing values"

Even if your data falls within your specified limits (e.g. c(0, 335)), adding a geom_jitter() statement could push some points outside those limits, producing the same error message.

library(ggplot2)

range(mtcars$hp)
#> [1]  52 335

# No jitter -- no error message
ggplot(mtcars, aes(mpg, hp)) + 
    geom_point() +
    scale_y_continuous(limits=c(0,335))


# Jitter is too large -- this generates the error message
ggplot(mtcars, aes(mpg, hp)) + 
    geom_point() +
    geom_jitter(position = position_jitter(w = 0.2, h = 0.2)) +
    scale_y_continuous(limits=c(0,335))
#> Warning: Removed 1 rows containing missing values (geom_point).

Created on 2020-08-24 by the reprex package (v0.3.0)

What is tail call optimization?

The recursive function approach has a problem. It builds up a call stack of size O(n), which makes our total memory cost O(n). This makes it vulnerable to a stack overflow error, where the call stack gets too big and runs out of space.

Tail call optimization (TCO) scheme. Where it can optimize recursive functions to avoid building up a tall call stack and hence saves the memory cost.

There are many languages who are doing TCO like (JavaScript, Ruby and few C) whereas Python and Java do not do TCO.

JavaScript language has confirmed using :) http://2ality.com/2015/06/tail-call-optimization.html

Adding link a href to an element using css

You cannot simply add a link using CSS. CSS is used for styling.

You can style your using CSS.

If you want to give a link dynamically to then I will advice you to use jQuery or Javascript.

You can accomplish that very easily using jQuery.

I have done a sample for you. You can refer that.

URL : http://jsfiddle.net/zyk9w/

$('#link').attr('href','http://www.google.com');

This single line will do the trick.

invalid_grant trying to get oAuth token from google

For me the issues was I had multiple clients in my project and I am pretty sure this is perfectly alright, but I deleted all the client for that project and created a new one and all started working for me ( Got this idea fro WP_SMTP plugin help support forum) I am not able to find out that link for reference

How do I get out of 'screen' without typing 'exit'?

  • Ctrl + A and then Ctrl+D. Doing this will detach you from the screen session which you can later resume by doing screen -r.

  • You can also do: Ctrl+A then type :. This will put you in screen command mode. Type the command detach to be detached from the running screen session.

need to add a class to an element

Try using setAttribute on the result:

result.setAttribute("class","red"); 

What does "#include <iostream>" do?

That is a C++ standard library header file for input output streams. It includes functionality to read and write from streams. You only need to include it if you wish to use streams.

How do I start a program with arguments when debugging?

I would suggest using the directives like the following:

        static void Main(string[] args)
        {
#if DEBUG
            args = new[] { "A" };
#endif

            Console.WriteLine(args[0]);
        }

Good luck!

iPhone UILabel text soft shadow

This like a trick,

UILabel *customLabel = [[UILabel alloc] init];

UIColor *color = [UIColor blueColor];
customLabel.layer.shadowColor = [color CGColor];
customLabel.layer.shadowRadius = 5.0f;
customLabel.layer.shadowOpacity = 1;
customLabel.layer.shadowOffset = CGSizeZero;
customLabel.layer.masksToBounds = NO;

How to change css property using javascript

This is really easy using jQuery.

For instance:

$(".left").mouseover(function(){$(".left1").show()});
$(".left").mouseout(function(){$(".left1").hide()});

I've update your fiddle: http://jsfiddle.net/TqDe9/2/

React onClick and preventDefault() link refresh/redirect?

none of these methods worked for me, so I just solved this with CSS:

.upvotes:before {
   content:"";
   float: left;
   width: 100%;
   height: 100%;
   position: absolute;
   left: 0;
   top: 0;
}

how to assign a block of html code to a javascript variable

 var test = "<div class='saved' >"+
 "<div >test.test</div> <div class='remove'>[Remove]</div></div>";

You can add "\n" if you require line-break.

How to make a div with no content have a width?

Either use padding , height or &nbsp for width to take effect with empty div

EDIT:

Non zero min-height also works great

Last Run Date on a Stored Procedure in SQL Server

If you enable Query Store on SQL Server 2016 or newer you can use the following query to get last SP execution. The history depends on the Query Store Configuration.

SELECT 
      ObjectName = '[' + s.name + '].[' + o.Name  + ']'
    , LastModificationDate  = MAX(o.modify_date)
    , LastExecutionTime     = MAX(q.last_execution_time)
FROM sys.query_store_query q 
    INNER JOIN sys.objects o
        ON q.object_id = o.object_id
    INNER JOIN sys.schemas s
        ON o.schema_id = s.schema_id
WHERE o.type IN ('P')
GROUP BY o.name , + s.name 

replace anchor text with jquery

function liReplace(replacement) {
    $(".dropit-submenu li").each(function() {
        var t = $(this);
        t.html(t.html().replace(replacement, "*" + replacement + "*"));
        t.children(":first").html(t.children(":first").html().replace(replacement, "*" +` `replacement + "*"));
        t.children(":first").html(t.children(":first").html().replace(replacement + " ", ""));
        alert(t.children(":first").text());
    });
}
  • First code find a title replace t.html(t.html()
  • Second code a text replace t.children(":first")

Sample <a title="alpc" href="#">alpc</a>

Is there a Wikipedia API?

Wikipedia is built on MediaWiki, and here's the MediaWiki API.

Selectors in Objective-C?

Don't think of the colon as part of the function name, think of it as a separator, if you don't have anything to separate (no value to go with the function) then you don't need it.

I'm not sure why but all this OO stuff seems to be foreign to Apple developers. I would strongly suggest grabbing Visual Studio Express and playing around with that too. Not because one is better than the other, just it's a good way to look at the design issues and ways of thinking.

Like

introspection = reflection
+ before functions/properties = static
- = instance level

It's always good to look at a problem in different ways and programming is the ultimate puzzle.

How to use a client certificate to authenticate and authorize in a Web API

Update:

Example from Microsoft:

https://docs.microsoft.com/en-us/azure/app-service/app-service-web-configure-tls-mutual-auth#special-considerations-for-certificate-validation

Original

This is how I got client certification working and checking that a specific Root CA had issued it as well as it being a specific certificate.

First I edited <src>\.vs\config\applicationhost.config and made this change: <section name="access" overrideModeDefault="Allow" />

This allows me to edit <system.webServer> in web.config and add the following lines which will require a client certification in IIS Express. Note: I edited this for development purposes, do not allow overrides in production.

For production follow a guide like this to set up the IIS:

https://medium.com/@hafizmohammedg/configuring-client-certificates-on-iis-95aef4174ddb

web.config:

<security>
  <access sslFlags="Ssl,SslNegotiateCert,SslRequireCert" />
</security>

API Controller:

[RequireSpecificCert]
public class ValuesController : ApiController
{
    // GET api/values
    public IHttpActionResult Get()
    {
        return Ok("It works!");
    }
}

Attribute:

public class RequireSpecificCertAttribute : AuthorizationFilterAttribute
{
    public override void OnAuthorization(HttpActionContext actionContext)
    {
        if (actionContext.Request.RequestUri.Scheme != Uri.UriSchemeHttps)
        {
            actionContext.Response = new HttpResponseMessage(System.Net.HttpStatusCode.Forbidden)
            {
                ReasonPhrase = "HTTPS Required"
            };
        }
        else
        {
            X509Certificate2 cert = actionContext.Request.GetClientCertificate();
            if (cert == null)
            {
                actionContext.Response = new HttpResponseMessage(System.Net.HttpStatusCode.Forbidden)
                {
                    ReasonPhrase = "Client Certificate Required"
                };

            }
            else
            {
                X509Chain chain = new X509Chain();

                //Needed because the error "The revocation function was unable to check revocation for the certificate" happened to me otherwise
                chain.ChainPolicy = new X509ChainPolicy()
                {
                    RevocationMode = X509RevocationMode.NoCheck,
                };
                try
                {
                    var chainBuilt = chain.Build(cert);
                    Debug.WriteLine(string.Format("Chain building status: {0}", chainBuilt));

                    var validCert = CheckCertificate(chain, cert);

                    if (chainBuilt == false || validCert == false)
                    {
                        actionContext.Response = new HttpResponseMessage(System.Net.HttpStatusCode.Forbidden)
                        {
                            ReasonPhrase = "Client Certificate not valid"
                        };
                        foreach (X509ChainStatus chainStatus in chain.ChainStatus)
                        {
                            Debug.WriteLine(string.Format("Chain error: {0} {1}", chainStatus.Status, chainStatus.StatusInformation));
                        }
                    }
                }
                catch (Exception ex)
                {
                    Debug.WriteLine(ex.ToString());
                }
            }

            base.OnAuthorization(actionContext);
        }
    }

    private bool CheckCertificate(X509Chain chain, X509Certificate2 cert)
    {
        var rootThumbprint = WebConfigurationManager.AppSettings["rootThumbprint"].ToUpper().Replace(" ", string.Empty);

        var clientThumbprint = WebConfigurationManager.AppSettings["clientThumbprint"].ToUpper().Replace(" ", string.Empty);

        //Check that the certificate have been issued by a specific Root Certificate
        var validRoot = chain.ChainElements.Cast<X509ChainElement>().Any(x => x.Certificate.Thumbprint.Equals(rootThumbprint, StringComparison.InvariantCultureIgnoreCase));

        //Check that the certificate thumbprint matches our expected thumbprint
        var validCert = cert.Thumbprint.Equals(clientThumbprint, StringComparison.InvariantCultureIgnoreCase);

        return validRoot && validCert;
    }
}

Can then call the API with client certification like this, tested from another web project.

[RoutePrefix("api/certificatetest")]
public class CertificateTestController : ApiController
{

    public IHttpActionResult Get()
    {
        var handler = new WebRequestHandler();
        handler.ClientCertificateOptions = ClientCertificateOption.Manual;
        handler.ClientCertificates.Add(GetClientCert());
        handler.UseProxy = false;
        var client = new HttpClient(handler);
        var result = client.GetAsync("https://localhost:44331/api/values").GetAwaiter().GetResult();
        var resultString = result.Content.ReadAsStringAsync().GetAwaiter().GetResult();
        return Ok(resultString);
    }

    private static X509Certificate GetClientCert()
    {
        X509Store store = null;
        try
        {
            store = new X509Store(StoreName.My, StoreLocation.CurrentUser);
            store.Open(OpenFlags.OpenExistingOnly | OpenFlags.ReadOnly);

            var certificateSerialNumber= "?81 c6 62 0a 73 c7 b1 aa 41 06 a3 ce 62 83 ae 25".ToUpper().Replace(" ", string.Empty);

            //Does not work for some reason, could be culture related
            //var certs = store.Certificates.Find(X509FindType.FindBySerialNumber, certificateSerialNumber, true);

            //if (certs.Count == 1)
            //{
            //    var cert = certs[0];
            //    return cert;
            //}

            var cert = store.Certificates.Cast<X509Certificate>().FirstOrDefault(x => x.GetSerialNumberString().Equals(certificateSerialNumber, StringComparison.InvariantCultureIgnoreCase));

            return cert;
        }
        finally
        {
            store?.Close();
        }
    }
}

jQuery $.ajax request of dataType json will not retrieve data from PHP script

The $.ajax error function takes three arguments, not one:

error: function(xhr, status, thrown)

You need to dump the 2nd and 3rd parameters to find your cause, not the first one.

Best way to store date/time in mongodb

One datestamp is already in the _id object, representing insert time

So if the insert time is what you need, it's already there:

Login to mongodb shell

ubuntu@ip-10-0-1-223:~$ mongo 10.0.1.223
MongoDB shell version: 2.4.9
connecting to: 10.0.1.223/test

Create your database by inserting items

> db.penguins.insert({"penguin": "skipper"})
> db.penguins.insert({"penguin": "kowalski"})
> 

Lets make that database the one we are on now

> use penguins
switched to db penguins

Get the rows back:

> db.penguins.find()
{ "_id" : ObjectId("5498da1bf83a61f58ef6c6d5"), "penguin" : "skipper" }
{ "_id" : ObjectId("5498da28f83a61f58ef6c6d6"), "penguin" : "kowalski" }

Get each row in yyyy-MM-dd HH:mm:ss format:

> db.penguins.find().forEach(function (doc){ d = doc._id.getTimestamp(); print(d.getFullYear()+"-"+(d.getMonth()+1)+"-"+d.getDate() + " " + d.getHours() + ":" + d.getMinutes() + ":" + d.getSeconds()) })
2014-12-23 3:4:41
2014-12-23 3:4:53

If that last one-liner confuses you I have a walkthrough on how that works here: https://stackoverflow.com/a/27613766/445131

How can I view the allocation unit size of a NTFS partition in Vista?

Open an administrator command prompt, and do this command:

fsutil fsinfo ntfsinfo [your drive]

The Bytes Per Cluster is the equivalent of the allocation unit.

How to remove specific value from array using jQuery

//This prototype function allows you to remove even array from array
Array.prototype.remove = function(x) { 
    var i;
    for(i in this){
        if(this[i].toString() == x.toString()){
            this.splice(i,1)
        }
    }
}

Example of using

var arr = [1,2,[1,1], 'abc'];
arr.remove([1,1]);
console.log(arr) //[1, 2, 'abc']

var arr = [1,2,[1,1], 'abc'];
arr.remove(1);
console.log(arr) //[2, [1,1], 'abc']

var arr = [1,2,[1,1], 'abc'];
arr.remove('abc');
console.log(arr) //[1, 2, [1,1]]

To use this prototype function you need to paste it in your code. Then you can apply it to any array with 'dot notation':

someArr.remove('elem1')

Ant build failed: "Target "build..xml" does not exist"

Is this a problem Only when you run ant -d or ant -verbose, but works other times?

I noticed this error message line:

Could not load definitions from resource org/apache/tools/ant/antlib.xml. It could not be found.

The org/apache/tools/ant/antlib.xml file is embedded in the ant.jar. The ant command is really a shell script called ant or a batch script called ant.bat. If the environment variable ANT_HOME is not set, it will figure out where it's located by looking to see where the ant command itself is located.

Sometimes I've seen this where someone will move the ant shell/batch script to put it in their path, and have ANT_HOME either not set, or set incorrectly.

What platform are you on? Is this Windows or Unix/Linux/MacOS? If you're on Windows, check to see if %ANT_HOME% is set. If it is, make sure it's the right directory. Make sure you have set your PATH to include %ANT_HOME%/bin.

If you're on Unix, don't copy the ant shell script to an executable directory. Instead, make a symbolic link. I have a directory called /usr/local/bin where I put the command I want to override the commands in /bin and /usr/bin. My Ant is installed in /opt/ant-1.9.2, and I have a symbolic link from /opt/ant-1.9.2 to /opt/ant. Then, I have symbolic links from all commands in /opt/ant/bin to /usr/local/bin. The Ant shell script can detect the symbolic links and find the correct Ant HOME location.

Next, go to your Ant installation directory and look under the lib directory to make sure ant.jar is there, and that it contains org/apache/tools/ant/antlib.xml. You can use the jar tvf ant.jar command. The only thing I can emphasize is that you do have everything setup correctly. You have your Ant shell script in your PATH either because you included the bin directory of your Ant's HOME directory in your classpath, or (if you're on Unix/Linux/MacOS), you have that file symbolically linked to a directory in your PATH.

Make sure your JAVA_HOME is set correctly (on Unix, you can use the symbolic link trick to have the java command set it for you), and that you're using Java 1.5 or higher. Java 1.4 will no longer work with newer versions of Ant.

Also run ant -version and see what you get. You might get the same error as before which leads me to think you have something wrong.

Let me know what you find, and your OS, and I can give you directions on reinstalling Ant.

Get the string representation of a DOM node

I have wasted a lot of time figuring out what is wrong when I iterate through DOMElements with the code in the accepted answer. This is what worked for me, otherwise every second element disappears from the document:

_getGpxString: function(node) {
          clone = node.cloneNode(true);
          var tmp = document.createElement("div");
          tmp.appendChild(clone);
          return tmp.innerHTML;
        },

How can you have SharePoint Link Lists default to opening in a new window?

Under the Links Tab ==> Edit the URL Item ==> Under the URL (Type the Web address)- format the value as follows:

Example: if the URL = http://www.abc.com ==> then suffix the value with ==>

  • #openinnewwindow/,'" target="http://www.abc.com'

SO, the final value should read as ==> http://www.abc.com#openinnewwindow/,'" target="http://www.abc.com'

DONE ==> this will open the URL in New Window

How do I get the SQLSRV extension to work with PHP, since MSSQL is deprecated?

Download Microsoft Drivers for PHP for SQL Server. Extract the files and use one of:

File                             Thread Safe         VC Bulid
php_sqlsrv_53_nts_vc6.dll           No                  VC6
php_sqlsrv_53_nts_vc9.dll           No                  VC9
php_sqlsrv_53_ts_vc6.dll            Yes                 VC6
php_sqlsrv_53_ts_vc9.dll            Yes                 VC9

You can see the Thread Safety status in phpinfo().

Add the correct file to your ext directory and the following line to your php.ini:

extension=php_sqlsrv_53_*_vc*.dll

Use the filename of the file you used.

As Gordon already posted this is the new Extension from Microsoft and uses the sqlsrv_* API instead of mssql_*

Update:
On Linux you do not have the requisite drivers and neither the SQLSERV Extension.
Look at Connect to MS SQL Server from PHP on Linux? for a discussion on this.

In short you need to install FreeTDS and YES you need to use mssql_* functions on linux. see update 2

To simplify things in the long run I would recommend creating a wrapper class with requisite functions which use the appropriate API (sqlsrv_* or mssql_*) based on which extension is loaded.

Update 2: You do not need to use mssql_* functions on linux. You can connect to an ms sql server using PDO + ODBC + FreeTDS. On windows, the best performing method to connect is via PDO + ODBC + SQL Native Client since the PDO + SQLSRV driver can be incredibly slow.

How to close a GUI when I push a JButton?

See JFrame.setDefaultCloseOperation(DISPOSE_ON_CLOSE)1. You might also use EXIT_ON_CLOSE, but it is better to explicitly clean up any running threads, then when the last GUI element becomes invisible, the EDT & JRE will end.

The 'button' to invoke this operation is already on a frame.

  1. See this answer to How to best position Swing GUIs? for a demo. of the DISPOSE_ON_CLOSE functionality.

    The JRE will end after all 3 frames are closed by clicking the X button.

Setting Different Bar color in matplotlib Python

I assume you are using Series.plot() to plot your data. If you look at the docs for Series.plot() here:

http://pandas.pydata.org/pandas-docs/dev/generated/pandas.Series.plot.html

there is no color parameter listed where you might be able to set the colors for your bar graph.

However, the Series.plot() docs state the following at the end of the parameter list:

kwds : keywords
Options to pass to matplotlib plotting method

What that means is that when you specify the kind argument for Series.plot() as bar, Series.plot() will actually call matplotlib.pyplot.bar(), and matplotlib.pyplot.bar() will be sent all the extra keyword arguments that you specify at the end of the argument list for Series.plot().

If you examine the docs for the matplotlib.pyplot.bar() method here:

http://matplotlib.org/api/pyplot_api.html#matplotlib.pyplot.bar

..it also accepts keyword arguments at the end of it's parameter list, and if you peruse the list of recognized parameter names, one of them is color, which can be a sequence specifying the different colors for your bar graph.

Putting it all together, if you specify the color keyword argument at the end of your Series.plot() argument list, the keyword argument will be relayed to the matplotlib.pyplot.bar() method. Here is the proof:

import pandas as pd
import matplotlib.pyplot as plt

s = pd.Series(
    [5, 4, 4, 1, 12],
    index = ["AK", "AX", "GA", "SQ", "WN"]
)

#Set descriptions:
plt.title("Total Delay Incident Caused by Carrier")
plt.ylabel('Delay Incident')
plt.xlabel('Carrier')

#Set tick colors:
ax = plt.gca()
ax.tick_params(axis='x', colors='blue')
ax.tick_params(axis='y', colors='red')

#Plot the data:
my_colors = 'rgbkymc'  #red, green, blue, black, etc.

pd.Series.plot(
    s, 
    kind='bar', 
    color=my_colors,
)

plt.show()

enter image description here

Note that if there are more bars than colors in your sequence, the colors will repeat.

How to delete and recreate from scratch an existing EF Code First database

Since this question is gonna be clicked some day by new EF Core users and I find the top answers somewhat unnecessarily destructive, I will show you a way to start "fresh". Beware, this deletes all of your data.

  1. Delete all tables on your MS SQL server. Also delete the __EFMigrations table.
  2. Type dotnet ef database update
  3. EF Core will now recreate the database from zero up until your latest migration.

How to join entries in a set into one string?

I think you just have it backwards.

print ", ".join(set_3)

Share variables between files in Node.js?

If we need to share multiple variables use the below format

//module.js
   let name='foobar';
   let city='xyz';
   let company='companyName';

   module.exports={
    name,
    city,
    company
  }

Usage

  // main.js
    require('./modules');
    console.log(name); // print 'foobar'

AngularJS app.run() documentation?

Here's the calling order:

  1. app.config()
  2. app.run()
  3. directive's compile functions (if they are found in the dom)
  4. app.controller()
  5. directive's link functions (again, if found)

Here's a simple demo where you can watch each one executing (and experiment if you'd like).

From Angular's module docs:

Run blocks - get executed after the injector is created and are used to kickstart the application. Only instances and constants can be injected into run blocks. This is to prevent further system configuration during application run time.

Run blocks are the closest thing in Angular to the main method. A run block is the code which needs to run to kickstart the application. It is executed after all of the services have been configured and the injector has been created. Run blocks typically contain code which is hard to unit-test, and for this reason should be declared in isolated modules, so that they can be ignored in the unit-tests.

One situation where run blocks are used is during authentications.

How to get DateTime.Now() in YYYY-MM-DDThh:mm:ssTZD format using C#

Try this:

DateTime.Now.ToString("yyyy-MM-ddThh:mm:sszzz");

zzz is the timezone offset.

Pythonic way to check if a list is sorted or not

Lazy

from itertools import tee

def is_sorted(l):
    l1, l2 = tee(l)
    next(l2, None)
    return all(a <= b for a, b in zip(l1, l2))

NodeJS: How to decode base64 encoded string back to binary?

As of Node.js v6.0.0 using the constructor method has been deprecated and the following method should instead be used to construct a new buffer from a base64 encoded string:

var b64string = /* whatever */;
var buf = Buffer.from(b64string, 'base64'); // Ta-da

For Node.js v5.11.1 and below

Construct a new Buffer and pass 'base64' as the second argument:

var b64string = /* whatever */;
var buf = new Buffer(b64string, 'base64'); // Ta-da

If you want to be clean, you can check whether from exists :

if (typeof Buffer.from === "function") {
    // Node 5.10+
    buf = Buffer.from(b64string, 'base64'); // Ta-da
} else {
    // older Node versions, now deprecated
    buf = new Buffer(b64string, 'base64'); // Ta-da
}

PHP: How to remove specific element from an array?

I would prefer to use array_key_exists to search for keys in arrays like:

Array([0]=>'A',[1]=>'B',['key'=>'value'])

to find the specified effectively, since array_search and in_array() don't work here. And do removing stuff with unset().

I think it will help someone.

Passing dynamic javascript values using Url.action()

This answer might not be 100% relevant to the question. But it does address the problem. I found this simple way of achieving this requirement. Code goes below:

<a href="@Url.Action("Display", "Customer")?custId={{cust.Id}}"></a>

In the above example {{cust.Id}} is an AngularJS variable. However one can replace it with a JavaScript variable.

I haven't tried passing multiple variables using this method but I'm hopeful that also can be appended to the Url if required.

Why is String immutable in Java?

From the Security point of view we can use this practical example:

DBCursor makeConnection(String IP,String PORT,String USER,String PASS,String TABLE) {

    // if strings were mutable IP,PORT,USER,PASS can be changed by validate function
    Boolean validated = validate(IP,PORT,USER,PASS);

    // here we are not sure if IP, PORT, USER, PASS changed or not ??
    if (validated) {
         DBConnection conn = doConnection(IP,PORT,USER,PASS);
    }

    // rest of the code goes here ....
}

Android view pager with page indicator

I know this has already been answered, but for anybody looking for a simple, no-frills implementation of a ViewPager indicator, I've implemented one that I've open sourced. For anyone finding Jake Wharton's version a bit complex for their needs, have a look at https://github.com/jarrodrobins/SimpleViewPagerIndicator.

Search and replace a line in a file in Python

Using hamishmcn's answer as a template I was able to search for a line in a file that match my regex and replacing it with empty string.

import re 

fin = open("in.txt", 'r') # in file
fout = open("out.txt", 'w') # out file
for line in fin:
    p = re.compile('[-][0-9]*[.][0-9]*[,]|[-][0-9]*[,]') # pattern
    newline = p.sub('',line) # replace matching strings with empty string
    print newline
    fout.write(newline)
fin.close()
fout.close()

Detect when an HTML5 video finishes

Here is a simple approach which triggers when the video ends.

<html>
<body>

    <video id="myVideo" controls="controls">
        <source src="video.mp4" type="video/mp4">
        etc ...
    </video>

</body>
<script type='text/javascript'>

    document.getElementById('myVideo').addEventListener('ended', function(e) {

        alert('The End');

    })

</script>
</html> 

In the 'EventListener' line substitute the word 'ended' with 'pause' or 'play' to capture those events as well.

How do the likely/unlikely macros in the Linux kernel work and what is their benefit?

long __builtin_expect(long EXP, long C);

This construct tells the compiler that the expression EXP most likely will have the value C. The return value is EXP. __builtin_expect is meant to be used in an conditional expression. In almost all cases will it be used in the context of boolean expressions in which case it is much more convenient to define two helper macros:

#define unlikely(expr) __builtin_expect(!!(expr), 0)
#define likely(expr) __builtin_expect(!!(expr), 1)

These macros can then be used as in

if (likely(a > 1))

Reference: https://www.akkadia.org/drepper/cpumemory.pdf

Delete the 'first' record from a table in SQL Server, without a WHERE condition

Does this really make sense?
There is no "first" record in a relational database, you can only delete one random record.

Chrome net::ERR_INCOMPLETE_CHUNKED_ENCODING error

my guess is the server is not correctly handling the chunked transfer-encoding. It needs to terminal a chunked files with a terminal chunk to indicate the entire file has been transferred.So the code below maybe work:

echo "\n";
flush();
ob_flush();
exit(0);

Procedure or function !!! has too many arguments specified

Yet another cause of this error is when you are calling the stored procedure from code, and the parameter type in code does not match the type on the stored procedure.

How disable / remove android activity label and label bar?

with your toolbar you can solve that problem. use setTitle method.

 Toolbar   mToolbar = (Toolbar) findViewById(R.id.toolbar);
  mToolbar.setTitle("");
  setSupportActionBar(mToolbar);

super easy :)

SQL Server: Filter output of sp_who2

Slight improvement to Astander's answer. I like to put my criteria at top, and make it easier to reuse day to day:

DECLARE @Spid INT, @Status VARCHAR(MAX), @Login VARCHAR(MAX), @HostName VARCHAR(MAX), @BlkBy VARCHAR(MAX), @DBName VARCHAR(MAX), @Command VARCHAR(MAX), @CPUTime INT, @DiskIO INT, @LastBatch VARCHAR(MAX), @ProgramName VARCHAR(MAX), @SPID_1 INT, @REQUESTID INT

    --SET @SPID = 10
    --SET @Status = 'BACKGROUND'
    --SET @LOGIN = 'sa'
    --SET @HostName = 'MSSQL-1'
    --SET @BlkBy = 0
    --SET @DBName = 'master'
    --SET @Command = 'SELECT INTO'
    --SET @CPUTime = 1000
    --SET @DiskIO = 1000
    --SET @LastBatch = '10/24 10:00:00'
    --SET @ProgramName = 'Microsoft SQL Server Management Studio - Query'
    --SET @SPID_1 = 10
    --SET @REQUESTID = 0

    SET NOCOUNT ON 
    DECLARE @Table TABLE(
            SPID INT,
            Status VARCHAR(MAX),
            LOGIN VARCHAR(MAX),
            HostName VARCHAR(MAX),
            BlkBy VARCHAR(MAX),
            DBName VARCHAR(MAX),
            Command VARCHAR(MAX),
            CPUTime INT,
            DiskIO INT,
            LastBatch VARCHAR(MAX),
            ProgramName VARCHAR(MAX),
            SPID_1 INT,
            REQUESTID INT
    )
    INSERT INTO @Table EXEC sp_who2
    SET NOCOUNT OFF
    SELECT  *
    FROM    @Table
    WHERE
    (@Spid IS NULL OR SPID = @Spid)
    AND (@Status IS NULL OR Status = @Status)
    AND (@Login IS NULL OR Login = @Login)
    AND (@HostName IS NULL OR HostName = @HostName)
    AND (@BlkBy IS NULL OR BlkBy = @BlkBy)
    AND (@DBName IS NULL OR DBName = @DBName)
    AND (@Command IS NULL OR Command = @Command)
    AND (@CPUTime IS NULL OR CPUTime >= @CPUTime)
    AND (@DiskIO IS NULL OR DiskIO >= @DiskIO)
    AND (@LastBatch IS NULL OR LastBatch >= @LastBatch)
    AND (@ProgramName IS NULL OR ProgramName = @ProgramName)
    AND (@SPID_1 IS NULL OR SPID_1 = @SPID_1)
    AND (@REQUESTID IS NULL OR REQUESTID = @REQUESTID)

Convert hex string to int

This is the right answer:

myPassedColor = "#ffff8c85" int colorInt = Color.parseColor(myPassedColor)

What is the difference between LATERAL and a subquery in PostgreSQL?

First, Lateral and Cross Apply is same thing. Therefore you may also read about Cross Apply. Since it was implemented in SQL Server for ages, you will find more information about it then Lateral.

Second, according to my understanding, there is nothing you can not do using subquery instead of using lateral. But:

Consider following query.

Select A.*
, (Select B.Column1 from B where B.Fk1 = A.PK and Limit 1)
, (Select B.Column2 from B where B.Fk1 = A.PK and Limit 1)
FROM A 

You can use lateral in this condition.

Select A.*
, x.Column1
, x.Column2
FROM A LEFT JOIN LATERAL (
  Select B.Column1,B.Column2,B.Fk1 from B  Limit 1
) x ON X.Fk1 = A.PK

In this query you can not use normal join, due to limit clause. Lateral or Cross Apply can be used when there is not simple join condition.

There are more usages for lateral or cross apply but this is most common one I found.

Access item in a list of lists

for l in list1:
    val = 50 - l[0] + l[1] - l[2]
    print "val:", val

Loop through list and do operation on the sublist as you wanted.

How do I fix "The expression of type List needs unchecked conversion...'?

This is a common problem when dealing with pre-Java 5 APIs. To automate the solution from erickson, you can create the following generic method:

public static <T> List<T> castList(Class<? extends T> clazz, Collection<?> c) {
    List<T> r = new ArrayList<T>(c.size());
    for(Object o: c)
      r.add(clazz.cast(o));
    return r;
}

This allows you to do:

List<SyndEntry> entries = castList(SyndEntry.class, sf.getEntries());

Because this solution checks that the elements indeed have the correct element type by means of a cast, it is safe, and does not require SuppressWarnings.

Variable used in lambda expression should be final or effectively final

A variable used in lambda expression should be a final or effectively final, but you can assign a value to a final one element array.

private TimeZone extractCalendarTimeZoneComponent(Calendar cal, TimeZone calTz) {
    try {
        TimeZone calTzLocal[] = new TimeZone[1];
        calTzLocal[0] = calTz;
        cal.getComponents().get("VTIMEZONE").forEach(component -> {
            TimeZone v = component;
            v.getTimeZoneId();
            if (calTzLocal[0] == null) {
                calTzLocal[0] = TimeZone.getTimeZone(v.getTimeZoneId().getValue());
            }
        });
    } catch (Exception e) {
        log.warn("Unable to determine ical timezone", e);
    }
    return null;
}

How to compare a local git branch with its remote branch?

Setup

git config alias.udiff 'diff @{u}'

Diffing HEAD with HEAD@{upstream}

git fetch  # Do this if you want to compare with the network state of upstream; if the current local state is enough, you can skip this
git udiff

Diffing with an Arbitrary Remote Branch

This answers the question in your heading ("its remote"); if you want to diff against "a remote" (that isn't configured as the upstream for the branch), you need to target it directly. You can see all remote branches with the following:

git branch -r

You can see all configured remotes with the following:

git remote show

You can see the branch/tracking configuration for a single remote (e.g. origin) as follows:

git remote show origin

Once you determine the appropriate origin branch, just do a normal diff :)

git diff [MY_LOCAL] MY_REMOTE_BRANCH

Export to CSV using MVC, C# and jQuery

What happens if you get rid of the stringwriter:

        Response.Clear();
        Response.AddHeader("Content-Disposition", "attachment; filename=adressenbestand.csv");
        Response.ContentType = "text/csv";
        //write the header
        Response.Write(String.Format("{0},{1},{2},{3}", CMSMessages.EmailAddress, CMSMessages.Gender, CMSMessages.FirstName, CMSMessages.LastName));

        //write every subscriber to the file
        var resourceManager = new ResourceManager(typeof(CMSMessages));
        foreach (var record in filterRecords.Select(x => x.First().Subscriber))
        {
            Response.Write(String.Format("{0},{1},{2},{3}", record.EmailAddress, record.Gender.HasValue ? resourceManager.GetString(record.Gender.ToString()) : "", record.FirstName, record.LastName));
        }

        Response.End();

Find all files with name containing string

The -maxdepth option should be before the -name option, like below.,

find . -maxdepth 1 -name "string" -print

Convert integer into its character equivalent, where 0 => a, 1 => b, etc

Assuming you want uppercase case letters:

function numberToLetter(num){
        var alf={
            '0': 'A', '1': 'B', '2': 'C', '3': 'D', '4': 'E', '5': 'F', '6': 'G'
        };
        if(num.length== 1) return alf[num] || ' ';
        return num.split('').map(numberToLetter);
    }

Example:

numberToLetter('023') is ["A", "C", "D"]

numberToLetter('5') is "F"

number to letter function

Angular2 multiple router-outlet in the same template

yes you can, but you need to use aux routing. you will need to give a name to your router-outlet:

<router-outlet name="auxPathName"></router-outlet>

and setup your route config:

@RouteConfig([
  {path:'/', name: 'RetularPath', component: OneComponent, useAsDefault: true},
  {aux:'/auxRoute', name: 'AuxPath', component: SecondComponent}
])

Check out this example, and also this video.


Update for RC.5 Aux routes has changed a bit: in your router outlet use a name:

<router-outlet name="aux">

In your router config:

{path: '/auxRouter', component: secondComponentComponent, outlet: 'aux'}

Quickest way to compare two generic lists for differences

using System.Collections.Generic;
using System.Linq;

namespace YourProject.Extensions
{
    public static class ListExtensions
    {
        public static bool SetwiseEquivalentTo<T>(this List<T> list, List<T> other)
            where T: IEquatable<T>
        {
            if (list.Except(other).Any())
                return false;
            if (other.Except(list).Any())
                return false;
            return true;
        }
    }
}

Sometimes you only need to know if two lists are different, and not what those differences are. In that case, consider adding this extension method to your project. Note that your listed objects should implement IEquatable!

Usage:

public sealed class Car : IEquatable<Car>
{
    public Price Price { get; }
    public List<Component> Components { get; }

    ...
    public override bool Equals(object obj)
        => obj is Car other && Equals(other);

    public bool Equals(Car other)
        => Price == other.Price
            && Components.SetwiseEquivalentTo(other.Components);

    public override int GetHashCode()
        => Components.Aggregate(
            Price.GetHashCode(),
            (code, next) => code ^ next.GetHashCode()); // Bitwise XOR
}

Whatever the Component class is, the methods shown here for Car should be implemented almost identically.

It's very important to note how we've written GetHashCode. In order to properly implement IEquatable, Equals and GetHashCode must operate on the instance's properties in a logically compatible way.

Two lists with the same contents are still different objects, and will produce different hash codes. Since we want these two lists to be treated as equal, we must let GetHashCode produce the same value for each of them. We can accomplish this by delegating the hashcode to every element in the list, and using the standard bitwise XOR to combine them all. XOR is order-agnostic, so it doesn't matter if the lists are sorted differently. It only matters that they contain nothing but equivalent members.

Note: the strange name is to imply the fact that the method does not consider the order of the elements in the list. If you do care about the order of the elements in the list, this method is not for you!

How to switch between python 2.7 to python 3 from command line?

In case you have both python 2 and 3 in your path, you can move up the Python27 folder in your path, so it search and executes python 2 first.

Is there any "font smoothing" in Google Chrome?

Status of the issue, June 2014: Fixed with Chrome 37

Finally, the Chrome team will release a fix for this issue with Chrome 37 which will be released to public in July 2014. See example comparison of current stable Chrome 35 and latest Chrome 37 (early development preview) here:

enter image description here

Status of the issue, December 2013

1.) There is NO proper solution when loading fonts via @import, <link href= or Google's webfont.js. The problem is that Chrome simply requests .woff files from Google's API which render horribly. Surprisingly all other font file types render beautifully. However, there are some CSS tricks that will "smoothen" the rendered font a little bit, you'll find the workaround(s) deeper in this answer.

2.) There IS a real solution for this when self-hosting the fonts, first posted by Jaime Fernandez in another answer on this Stackoverflow page, which fixes this issue by loading web fonts in a special order. I would feel bad to simply copy his excellent answer, so please have a look there. There is also an (unproven) solution that recommends using only TTF/OTF fonts as they are now supported by nearly all browsers.

3.) The Google Chrome developer team works on that issue. As there have been several huge changes in the rendering engine there's obviously something in progress.

I've written a large blog post on that issue, feel free to have a look: How to fix the ugly font rendering in Google Chrome

Reproduceable examples

See how the example from the initial question look today, in Chrome 29:

POSITIVE EXAMPLE:

Left: Firefox 23, right: Chrome 29

enter image description here

POSITIVE EXAMPLE:

Top: Firefox 23, bottom: Chrome 29

enter image description here

NEGATIVE EXAMPLE: Chrome 30

enter image description here

NEGATIVE EXAMPLE: Chrome 29

enter image description here

Solution

Fixing the above screenshot with -webkit-text-stroke:

enter image description here

First row is default, second has:

-webkit-text-stroke: 0.3px;

Third row has:

-webkit-text-stroke: 0.6px;

So, the way to fix those fonts is simply giving them

-webkit-text-stroke: 0.Xpx;

or the RGBa syntax (by nezroy, found in the comments! Thanks!)

-webkit-text-stroke: 1px rgba(0,0,0,0.1)

There's also an outdated possibility: Give the text a simple (fake) shadow:

text-shadow: #fff 0px 1px 1px;

RGBa solution (found in Jasper Espejo's blog):

text-shadow: 0 0 1px rgba(51,51,51,0.2);

I made a blog post on this:

If you want to be updated on this issue, have a look on the according blog post: How to fix the ugly font rendering in Google Chrome. I'll post news if there're news on this.

My original answer:

This is a big bug in Google Chrome and the Google Chrome Team does know about this, see the official bug report here. Currently, in May 2013, even 11 months after the bug was reported, it's not solved. It's a strange thing that the only browser that messes up Google Webfonts is Google's own browser Chrome (!). But there's a simple workaround that will fix the problem, please see below for the solution.

STATEMENT FROM GOOGLE CHROME DEVELOPMENT TEAM, MAY 2013

Official statement in the bug report comments:

Our Windows font rendering is actively being worked on. ... We hope to have something within a milestone or two that developers can start playing with. How fast it goes to stable is, as always, all about how fast we can root out and burn down any regressions.

Get value of a merged cell of an excel from its cell address in vba

Even if it is really discouraged to use merge cells in Excel (use Center Across Selection for instance if needed), the cell that "contains" the value is the one on the top left (at least, that's a way to express it).

Hence, you can get the value of merged cells in range B4:B11 in several ways:

  • Range("B4").Value
  • Range("B4:B11").Cells(1).Value
  • Range("B4:B11").Cells(1,1).Value

You can also note that all the other cells have no value in them. While debugging, you can see that the value is empty.

Also note that Range("B4:B11").Value won't work (raises an execution error number 13 if you try to Debug.Print it) because it returns an array.

Unique on a dataframe with only selected columns

Using unique():

dat <- data.frame(id=c(1,1,3),id2=c(1,1,4),somevalue=c("x","y","z"))    
dat[row.names(unique(dat[,c("id", "id2")])),]

Using :focus to style outer div?

Simple use JQuery.

_x000D_
_x000D_
$(document).ready(function() {_x000D_
  $("div .FormRow").focusin(function() {_x000D_
    $(this).css("background-color", "#FFFFCC");_x000D_
    $(this).css("border", "3px solid #555");_x000D_
  });_x000D_
  $("div .FormRow").focusout(function() {_x000D_
    $(this).css("background-color", "#FFFFFF");_x000D_
    $(this).css("border", "0px solid #555");_x000D_
  });_x000D_
});
_x000D_
    .FormRow {_x000D_
      padding: 10px;_x000D_
    }
_x000D_
<html>_x000D_
_x000D_
<head>_x000D_
  <script src="http://ajax.googleapis.com/ajax/libs/jquery/1.11.1/jquery.min.js"></script>_x000D_
</head>_x000D_
_x000D_
<body>_x000D_
  <div style="border: 0px solid black;padding:10px;">_x000D_
    <div class="FormRow">_x000D_
      First Name:_x000D_
      <input type="text">_x000D_
      <br>_x000D_
    </div>_x000D_
    <div class="FormRow">_x000D_
      Last Name:_x000D_
      <input type="text">_x000D_
    </div>_x000D_
  </div>_x000D_
_x000D_
  <ul>_x000D_
    <li><strong><em>Click an input field to get focus.</em></strong>_x000D_
    </li>_x000D_
    <li><strong><em>Click outside an input field to lose focus.</em></strong>_x000D_
    </li>_x000D_
  </ul>_x000D_
</body>_x000D_
_x000D_
</html>
_x000D_
_x000D_
_x000D_

Export database schema into SQL file

i wrote this sp to create automatically the schema with all things, pk, fk, partitions, constraints... I wrote it to run in same sp.

IMPORTANT!! before exec

    create type TableType as table (ObjectID int)

here the SP:

create PROCEDURE [dbo].[util_ScriptTable] 
     @DBName SYSNAME
    ,@schema sysname
    ,@TableName SYSNAME
    ,@IncludeConstraints BIT = 1
    ,@IncludeIndexes BIT = 1
    ,@NewTableSchema sysname
    ,@NewTableName SYSNAME = NULL
    ,@UseSystemDataTypes BIT = 0
    ,@script varchar(max) output
AS 
BEGIN try
    if not exists (select * from sys.types where name = 'TableType')
        create type TableType as table (ObjectID int)--drop type TableType
    
    declare @sql nvarchar(max)
  
    DECLARE @MainDefinition TABLE (FieldValue VARCHAR(200))
    --DECLARE @DBName SYSNAME
    DECLARE @ClusteredPK BIT
    DECLARE @TableSchema NVARCHAR(255)
    
    --SET @DBName = DB_NAME(DB_ID())
    SELECT @TableName = name FROM sysobjects WHERE id = OBJECT_ID(@TableName)
    
    DECLARE @ShowFields TABLE (FieldID INT IDENTITY(1,1)
                                        ,DatabaseName VARCHAR(100)
                                        ,TableOwner VARCHAR(100)
                                        ,TableName VARCHAR(100)
                                        ,FieldName VARCHAR(100)
                                        ,ColumnPosition INT
                                        ,ColumnDefaultValue VARCHAR(100)
                                        ,ColumnDefaultName VARCHAR(100)
                                        ,IsNullable BIT
                                        ,DataType VARCHAR(100)
                                        ,MaxLength varchar(10)
                                        ,NumericPrecision INT
                                        ,NumericScale INT
                                        ,DomainName VARCHAR(100)
                                        ,FieldListingName VARCHAR(110)
                                        ,FieldDefinition CHAR(1)
                                        ,IdentityColumn BIT
                                        ,IdentitySeed INT
                                        ,IdentityIncrement INT
                                        ,IsCharColumn BIT 
                                        ,IsComputed varchar(255))
    
    DECLARE @HoldingArea TABLE(FldID SMALLINT IDENTITY(1,1)
                                        ,Flds VARCHAR(4000)
                                        ,FldValue CHAR(1) DEFAULT(0))
    
    DECLARE @PKObjectID TABLE(ObjectID INT)
    
    DECLARE @Uniques TABLE(ObjectID INT)
    
    DECLARE @HoldingAreaValues TABLE(FldID SMALLINT IDENTITY(1,1)
                                                ,Flds VARCHAR(4000)
                                                ,FldValue CHAR(1) DEFAULT(0))
    
    DECLARE @Definition TABLE(DefinitionID SMALLINT IDENTITY(1,1)
                                        ,FieldValue VARCHAR(200))

  
  set @sql=
  '
  use '+@DBName+'
  SELECT distinct DB_NAME()
            ,inf.TABLE_SCHEMA
            ,inf.TABLE_NAME
            ,''[''+inf.COLUMN_NAME+'']'' as COLUMN_NAME
            ,CAST(inf.ORDINAL_POSITION AS INT)
            ,inf.COLUMN_DEFAULT
            ,dobj.name AS ColumnDefaultName
            ,CASE WHEN inf.IS_NULLABLE = ''YES'' THEN 1 ELSE 0 END
            ,inf.DATA_TYPE
            ,case inf.CHARACTER_MAXIMUM_LENGTH when -1 then ''max'' else CAST(inf.CHARACTER_MAXIMUM_LENGTH AS varchar) end--CAST(CHARACTER_MAXIMUM_LENGTH AS INT)
            ,CAST(inf.NUMERIC_PRECISION AS INT)
            ,CAST(inf.NUMERIC_SCALE AS INT)
            ,inf.DOMAIN_NAME
            ,inf.COLUMN_NAME + '',''
            ,'''' AS FieldDefinition
            --caso di viste, dà come campo identity ma nn dà i valori, quindi lo ignoro
            ,CASE WHEN ic.object_id IS not NULL and ic.seed_value is not null THEN 1 ELSE 0 END AS IdentityColumn--CASE WHEN ic.object_id IS NULL THEN 0 ELSE 1 END AS IdentityColumn
            ,CAST(ISNULL(ic.seed_value,0) AS INT) AS IdentitySeed
            ,CAST(ISNULL(ic.increment_value,0) AS INT) AS IdentityIncrement
            ,CASE WHEN c.collation_name IS NOT NULL THEN 1 ELSE 0 END AS IsCharColumn 
            ,cc.definition 
            from (select schema_id,object_id,name from sys.views union all select schema_id,object_id,name from sys.tables)t
                --sys.tables t
            join sys.schemas s on t.schema_id=s.schema_id
            JOIN sys.columns c ON  t.object_id=c.object_id --AND s.schema_id=c.schema_id
            LEFT JOIN sys.identity_columns ic ON t.object_id=ic.object_id AND c.column_id=ic.column_id
            left JOIN sys.types st ON st.system_type_id=c.system_type_id and st.principal_id=t.object_id--COALESCE(c.DOMAIN_NAME,c.DATA_TYPE) = st.name
            LEFT OUTER JOIN sys.objects dobj ON dobj.object_id = c.default_object_id AND dobj.type = ''D''
            left join sys.computed_columns cc on t.object_id=cc.object_id and c.column_id=cc.column_id
            join INFORMATION_SCHEMA.COLUMNS inf on t.name=inf.TABLE_NAME
                                               and s.name=inf.TABLE_SCHEMA
                                               and c.name=inf.COLUMN_NAME
    WHERE inf.TABLE_NAME = @TableName and inf.TABLE_SCHEMA=@schema 
    ORDER BY inf.ORDINAL_POSITION
    '
    
  print @sql
  INSERT INTO @ShowFields( DatabaseName
                                    ,TableOwner
                                    ,TableName
                                    ,FieldName
                                    ,ColumnPosition
                                    ,ColumnDefaultValue
                                    ,ColumnDefaultName
                                    ,IsNullable
                                    ,DataType
                                    ,MaxLength
                                    ,NumericPrecision
                                    ,NumericScale
                                    ,DomainName
                                    ,FieldListingName
                                    ,FieldDefinition
                                    ,IdentityColumn
                                    ,IdentitySeed
                                    ,IdentityIncrement
                                    ,IsCharColumn
                                    ,IsComputed)
                                    
    exec sp_executesql @sql,
                       N'@TableName varchar(50),@schema varchar(50)',
                       @TableName=@TableName,@schema=@schema            
    /*
    SELECT @DBName--DB_NAME()
            ,TABLE_SCHEMA
            ,TABLE_NAME
            ,COLUMN_NAME
            ,CAST(ORDINAL_POSITION AS INT)
            ,COLUMN_DEFAULT
            ,dobj.name AS ColumnDefaultName
            ,CASE WHEN c.IS_NULLABLE = 'YES' THEN 1 ELSE 0 END
            ,DATA_TYPE
            ,CAST(CHARACTER_MAXIMUM_LENGTH AS INT)
            ,CAST(NUMERIC_PRECISION AS INT)
            ,CAST(NUMERIC_SCALE AS INT)
            ,DOMAIN_NAME
            ,COLUMN_NAME + ','
            ,'' AS FieldDefinition
            ,CASE WHEN ic.object_id IS NULL THEN 0 ELSE 1 END AS IdentityColumn
            ,CAST(ISNULL(ic.seed_value,0) AS INT) AS IdentitySeed
            ,CAST(ISNULL(ic.increment_value,0) AS INT) AS IdentityIncrement
            ,CASE WHEN st.collation_name IS NOT NULL THEN 1 ELSE 0 END AS IsCharColumn 
            FROM INFORMATION_SCHEMA.COLUMNS c
            JOIN sys.columns sc ON  c.TABLE_NAME = OBJECT_NAME(sc.object_id) AND c.COLUMN_NAME = sc.Name
            LEFT JOIN sys.identity_columns ic ON c.TABLE_NAME = OBJECT_NAME(ic.object_id) AND c.COLUMN_NAME = ic.Name
            JOIN sys.types st ON COALESCE(c.DOMAIN_NAME,c.DATA_TYPE) = st.name
            LEFT OUTER JOIN sys.objects dobj ON dobj.object_id = sc.default_object_id AND dobj.type = 'D'

    WHERE c.TABLE_NAME = @TableName
    ORDER BY c.TABLE_NAME, c.ORDINAL_POSITION
    */
    SELECT TOP 1 @TableSchema = TableOwner FROM @ShowFields
    
    INSERT INTO @HoldingArea (Flds) VALUES('(')
    
    INSERT INTO @Definition(FieldValue)VALUES('CREATE TABLE ' + CASE WHEN @NewTableName IS NOT NULL THEN @DBName + '.' + @NewTableSchema + '.' + @NewTableName ELSE @DBName + '.' + @TableSchema + '.' + @TableName END)
    INSERT INTO @Definition(FieldValue)VALUES('(')
    INSERT INTO @Definition(FieldValue)
    SELECT   CHAR(10) + FieldName + ' ' + 
        --CASE WHEN DomainName IS NOT NULL AND @UseSystemDataTypes = 0 THEN DomainName + CASE WHEN IsNullable = 1 THEN ' NULL ' ELSE ' NOT NULL ' END ELSE UPPER(DataType) +CASE WHEN IsCharColumn = 1 THEN '(' + CAST(MaxLength AS VARCHAR(10)) + ')' ELSE '' END +CASE WHEN IdentityColumn = 1 THEN ' IDENTITY(' + CAST(IdentitySeed AS VARCHAR(5))+ ',' + CAST(IdentityIncrement AS VARCHAR(5)) + ')' ELSE '' END +CASE WHEN IsNullable = 1 THEN ' NULL ' ELSE ' NOT NULL ' END +CASE WHEN ColumnDefaultName IS NOT NULL AND @IncludeConstraints = 1 THEN 'CONSTRAINT [' + ColumnDefaultName + '] DEFAULT' + UPPER(ColumnDefaultValue) ELSE '' END END + CASE WHEN FieldID = (SELECT MAX(FieldID) FROM @ShowFields) THEN '' ELSE ',' END 
        
        CASE WHEN DomainName IS NOT NULL AND @UseSystemDataTypes = 0 THEN DomainName + 
            CASe WHEN IsNullable = 1 THEN ' NULL ' 
            ELSE ' NOT NULL ' 
            END 
        ELSE 
            case when IsComputed is null then
                UPPER(DataType) +
                CASE WHEN IsCharColumn = 1 THEN '(' + CAST(MaxLength AS VARCHAR(10)) + ')' 
                ELSE 
                    CASE WHEN DataType = 'numeric' THEN '(' + CAST(NumericPrecision AS VARCHAR(10))+','+ CAST(NumericScale AS VARCHAR(10)) + ')' 
                    ELSE
                        CASE WHEN DataType = 'decimal' THEN '(' + CAST(NumericPrecision AS VARCHAR(10))+','+ CAST(NumericScale AS VARCHAR(10)) + ')' 
                        ELSE '' 
                        end  
                    end 
                END +
                CASE WHEN IdentityColumn = 1 THEN ' IDENTITY(' + CAST(IdentitySeed AS VARCHAR(5))+ ',' + CAST(IdentityIncrement AS VARCHAR(5)) + ')' 
                ELSE '' 
                END +
                CASE WHEN IsNullable = 1 THEN ' NULL ' 
                ELSE ' NOT NULL ' 
                END +
                CASE WHEN ColumnDefaultName IS NOT NULL AND @IncludeConstraints = 1 THEN 'CONSTRAINT [' + replace(ColumnDefaultName,@TableName,@NewTableName) + '] DEFAULT' + UPPER(ColumnDefaultValue) 
                ELSE '' 
                END 
            else
                ' as '+IsComputed+' '
            end
        END + 
        CASE WHEN FieldID = (SELECT MAX(FieldID) FROM @ShowFields) THEN '' 
        ELSE ',' 
        END 
        
    FROM    @ShowFields
    
    IF @IncludeConstraints = 1
        BEGIN    
        
        set @sql=
        '
        use '+@DBName+'
        SELECT  distinct  '',CONSTRAINT ['' + @NewTableName+''_''+replace(name,@TableName,'''') + ''] FOREIGN KEY ('' + ParentColumns + '') REFERENCES ['' + ReferencedObject + '']('' + ReferencedColumns + '')'' 
           FROM ( SELECT   ReferencedObject = OBJECT_NAME(fk.referenced_object_id), ParentObject = OBJECT_NAME(parent_object_id),fk.name
                ,   REVERSE(SUBSTRING(REVERSE((   SELECT cp.name + '',''   
                FROM   sys.foreign_key_columns fkc   
                JOIN sys.columns cp ON fkc.parent_object_id = cp.object_id AND fkc.parent_column_id = cp.column_id   
                WHERE fkc.constraint_object_id = fk.object_id   FOR XML PATH('''')   )), 2, 8000)) ParentColumns,   
                REVERSE(SUBSTRING(REVERSE((   SELECT cr.name + '',''   
                FROM   sys.foreign_key_columns fkc  
                JOIN sys.columns cr ON fkc.referenced_object_id = cr.object_id AND fkc.referenced_column_id = cr.column_id
                WHERE fkc.constraint_object_id = fk.object_id   FOR XML PATH('''')   )), 2, 8000)) ReferencedColumns   
                FROM sys.foreign_keys fk    
                    inner join sys.schemas s on fk.schema_id=s.schema_id and s.name=@schema) a    
            WHERE ParentObject = @TableName    
        '
        
        print @sql
        
        INSERT INTO @Definition(FieldValue)
        exec sp_executesql @sql,
                   N'@TableName varchar(50),@NewTableName varchar(50),@schema varchar(50)',
                       @TableName=@TableName,@NewTableName=@NewTableName,@schema=@schema
            /*
           SELECT    ',CONSTRAINT [' + name + '] FOREIGN KEY (' + ParentColumns + ') REFERENCES [' + ReferencedObject + '](' + ReferencedColumns + ')'  
           FROM ( SELECT   ReferencedObject = OBJECT_NAME(fk.referenced_object_id), ParentObject = OBJECT_NAME(parent_object_id),fk.name
                ,   REVERSE(SUBSTRING(REVERSE((   SELECT cp.name + ','   
                FROM   sys.foreign_key_columns fkc   
                JOIN sys.columns cp ON fkc.parent_object_id = cp.object_id AND fkc.parent_column_id = cp.column_id   
                WHERE fkc.constraint_object_id = fk.object_id   FOR XML PATH('')   )), 2, 8000)) ParentColumns,   
                REVERSE(SUBSTRING(REVERSE((   SELECT cr.name + ','   
                FROM   sys.foreign_key_columns fkc  
                JOIN sys.columns cr ON fkc.referenced_object_id = cr.object_id AND fkc.referenced_column_id = cr.column_id
                WHERE fkc.constraint_object_id = fk.object_id   FOR XML PATH('')   )), 2, 8000)) ReferencedColumns   
                FROM sys.foreign_keys fk    ) a    
            WHERE ParentObject = @TableName    
            */
            
            set @sql=
            '
            use '+@DBName+'
            SELECT distinct '',CONSTRAINT ['' + @NewTableName+''_''+replace(c.name,@TableName,'''') + ''] CHECK '' + definition 
            FROM sys.check_constraints c join sys.schemas s on c.schema_id=s.schema_id and s.name=@schema    
            WHERE OBJECT_NAME(parent_object_id) = @TableName
            '
            
            print @sql
            INSERT INTO @Definition(FieldValue) 
            exec sp_executesql @sql,
                               N'@TableName varchar(50),@NewTableName varchar(50),@schema varchar(50)',
                       @TableName=@TableName,@NewTableName=@NewTableName,@schema=@schema
            /*
            SELECT ',CONSTRAINT [' + name + '] CHECK ' + definition FROM sys.check_constraints    
            WHERE OBJECT_NAME(parent_object_id) = @TableName
            */
            
            set @sql=
            '
            use '+@DBName+'
            SELECT DISTINCT  PKObject = cco.object_id 
            FROM    sys.key_constraints cco    
            JOIN sys.index_columns cc ON cco.parent_object_id = cc.object_id AND cco.unique_index_id = cc.index_id    
            JOIN sys.indexes i ON cc.object_id = i.object_id AND cc.index_id = i.index_id
            join sys.schemas s on cco.schema_id=s.schema_id and s.name=@schema
            WHERE    OBJECT_NAME(parent_object_id) = @TableName    AND  i.type = 1 AND    is_primary_key = 1
            '
            print @sql
            INSERT INTO @PKObjectID(ObjectID) 
            exec sp_executesql @sql,
                               N'@TableName varchar(50),@schema varchar(50)',
                               @TableName=@TableName,@schema=@schema
            /*
            SELECT DISTINCT  PKObject = cco.object_id 
            FROM    sys.key_constraints cco    
            JOIN sys.index_columns cc ON cco.parent_object_id = cc.object_id AND cco.unique_index_id = cc.index_id    
            JOIN sys.indexes i ON cc.object_id = i.object_id AND cc.index_id = i.index_id
            WHERE    OBJECT_NAME(parent_object_id) = @TableName    AND  i.type = 1 AND    is_primary_key = 1
            */
            
            set @sql=
            '
            use '+@DBName+'
            SELECT DISTINCT    PKObject = cco.object_id
            FROM    sys.key_constraints cco   
            JOIN sys.index_columns cc ON cco.parent_object_id = cc.object_id AND cco.unique_index_id = cc.index_id  
            JOIN sys.indexes i ON cc.object_id = i.object_id AND cc.index_id = i.index_id
            join sys.schemas s on cco.schema_id=s.schema_id and s.name=@schema
            WHERE    OBJECT_NAME(parent_object_id) = @TableName AND  i.type = 2 AND    is_primary_key = 0 AND    is_unique_constraint = 1
            '
            print @sql
            INSERT INTO @Uniques(ObjectID)
            exec sp_executesql @sql,
                               N'@TableName varchar(50),@schema varchar(50)',
                               @TableName=@TableName,@schema=@schema
            /*
            SELECT DISTINCT    PKObject = cco.object_id
            FROM    sys.key_constraints cco   
            JOIN sys.index_columns cc ON cco.parent_object_id = cc.object_id AND cco.unique_index_id = cc.index_id  
            JOIN sys.indexes i ON cc.object_id = i.object_id AND cc.index_id = i.index_id
            WHERE    OBJECT_NAME(parent_object_id) = @TableName AND  i.type = 2 AND    is_primary_key = 0 AND    is_unique_constraint = 1
            */
            
            SET @ClusteredPK = CASE WHEN @@ROWCOUNT > 0 THEN 1 ELSE 0 END
            
            declare @t TableType
            insert @t select * from @PKObjectID
            declare @u TableType
            insert @u select * from @Uniques
            
            set @sql=
            '
            use '+@DBName+'
            SELECT distinct '',CONSTRAINT '' + @NewTableName+''_''+replace(cco.name,@TableName,'''') + CASE type WHEN ''PK'' THEN '' PRIMARY KEY '' + CASE WHEN pk.ObjectID IS NULL THEN '' NONCLUSTERED '' ELSE '' CLUSTERED '' END  WHEN ''UQ'' THEN '' UNIQUE '' END + CASE WHEN u.ObjectID IS NOT NULL THEN '' NONCLUSTERED '' ELSE '''' END 
            + ''(''+REVERSE(SUBSTRING(REVERSE(( SELECT   c.name +  + CASE WHEN cc.is_descending_key = 1 THEN '' DESC'' ELSE '' ASC'' END + '',''    
            FROM   sys.key_constraints ccok   
            LEFT JOIN sys.index_columns cc ON ccok.parent_object_id = cc.object_id AND cco.unique_index_id = cc.index_id
            LEFT JOIN sys.columns c ON cc.object_id = c.object_id AND cc.column_id = c.column_id 
            LEFT JOIN sys.indexes i ON cc.object_id = i.object_id AND cc.index_id = i.index_id  
            WHERE i.object_id = ccok.parent_object_id AND   ccok.object_id = cco.object_id    
            order by key_ordinal FOR XML PATH(''''))), 2, 8000)) + '')''
            FROM sys.key_constraints cco 
            inner join sys.schemas s on cco.schema_id=s.schema_id and s.name=@schema
            LEFT JOIN @U u ON cco.object_id = u.objectID
            LEFT JOIN @t pk ON cco.object_id = pk.ObjectID    
            WHERE    OBJECT_NAME(cco.parent_object_id) = @TableName 
            
            '
            
            print @sql
            INSERT INTO @Definition(FieldValue)
            exec sp_executesql @sql,
                               N'@TableName varchar(50),@NewTableName varchar(50),@schema varchar(50),@t TableType readonly,@u TableType readonly',
                               @TableName=@TableName,@NewTableName=@NewTableName,@schema=@schema,@t=@t,@u=@u
            
            /*
            SELECT ',CONSTRAINT ' + name + CASE type WHEN 'PK' THEN ' PRIMARY KEY ' + CASE WHEN pk.ObjectID IS NULL THEN ' NONCLUSTERED ' ELSE ' CLUSTERED ' END  WHEN 'UQ' THEN ' UNIQUE ' END + CASE WHEN u.ObjectID IS NOT NULL THEN ' NONCLUSTERED ' ELSE '' END 
            + '(' +REVERSE(SUBSTRING(REVERSE(( SELECT   c.name +  + CASE WHEN cc.is_descending_key = 1 THEN ' DESC' ELSE ' ASC' END + ','    
            FROM   sys.key_constraints ccok   
            LEFT JOIN sys.index_columns cc ON ccok.parent_object_id = cc.object_id AND cco.unique_index_id = cc.index_id
           LEFT JOIN sys.columns c ON cc.object_id = c.object_id AND cc.column_id = c.column_id 
           LEFT JOIN sys.indexes i ON cc.object_id = i.object_id AND cc.index_id = i.index_id  
           WHERE i.object_id = ccok.parent_object_id AND   ccok.object_id = cco.object_id    FOR XML PATH(''))), 2, 8000)) + ')'
           FROM sys.key_constraints cco 
           LEFT JOIN @PKObjectID pk ON cco.object_id = pk.ObjectID    
           LEFT JOIN @Uniques u ON cco.object_id = u.objectID
           WHERE    OBJECT_NAME(cco.parent_object_id) = @TableName 
           */
        END
           
        INSERT INTO @Definition(FieldValue) VALUES(')')
        
        set @sql=
        '
        use '+@DBName+'
        select '' on '' + d.name + ''([''+c.name+''])''
        from sys.tables t join sys.indexes i on(i.object_id = t.object_id and i.index_id < 2)
                          join sys.index_columns ic on(ic.partition_ordinal > 0 and ic.index_id = i.index_id and ic.object_id = t.object_id)
                          join sys.columns c on(c.object_id = ic.object_id and c.column_id = ic.column_id)
                          join sys.schemas s on t.schema_id=s.schema_id
                          join sys.data_spaces d on i.data_space_id=d.data_space_id
        where t.name=@TableName and s.name=@schema
        order by key_ordinal
        '
        
        print 'x'
        print @sql
        INSERT INTO @Definition(FieldValue) 
        exec sp_executesql @sql,
                           N'@TableName varchar(50),@schema varchar(50)',
                           @TableName=@TableName,@schema=@schema
             
        IF @IncludeIndexes = 1
        BEGIN
            set @sql=
            '
            use '+@DBName+'
            SELECT distinct '' CREATE '' + i.type_desc + '' INDEX ['' + replace(i.name COLLATE SQL_Latin1_General_CP1_CI_AS,@TableName,@NewTableName) + ''] ON '+@DBName+'.'+@NewTableSchema+'.'+@NewTableName+' ('' 
            +   REVERSE(SUBSTRING(REVERSE((   SELECT name + CASE WHEN sc.is_descending_key = 1 THEN '' DESC'' ELSE '' ASC'' END + '',''   
            FROM  sys.index_columns sc  
            JOIN sys.columns c ON sc.object_id = c.object_id AND sc.column_id = c.column_id   
            WHERE  t.name=@TableName AND  sc.object_id = i.object_id AND  sc.index_id = i.index_id   
                                         and is_included_column=0
            ORDER BY key_ordinal ASC   FOR XML PATH('''')    )), 2, 8000)) + '')''+
            ISNULL( '' include (''+REVERSE(SUBSTRING(REVERSE((   SELECT name + '',''   
            FROM  sys.index_columns sc  
            JOIN sys.columns c ON sc.object_id = c.object_id AND sc.column_id = c.column_id   
            WHERE  t.name=@TableName AND  sc.object_id = i.object_id AND  sc.index_id = i.index_id   
                                         and is_included_column=1
            ORDER BY key_ordinal ASC   FOR XML PATH('''')    )), 2, 8000))+'')'' ,'''')+''''    
            FROM sys.indexes i join sys.tables t on i.object_id=t.object_id
                               join sys.schemas s on t.schema_id=s.schema_id   
            AND CASE WHEN @ClusteredPK = 1 AND is_primary_key = 1 AND i.type = 1 THEN 0 ELSE 1 END = 1   AND is_unique_constraint = 0   AND is_primary_key = 0 
                where t.name=@TableName and s.name=@schema
            '
            
            print @sql
            INSERT INTO @Definition(FieldValue)    
            exec sp_executesql @sql,
                               N'@TableName varchar(50),@NewTableName varchar(50),@schema varchar(50), @ClusteredPK bit',
                               @TableName=@TableName,@NewTableName=@NewTableName,@schema=@schema,@ClusteredPK=@ClusteredPK
            
        END 
       
            /*
                
                SELECT   'CREATE ' + type_desc + ' INDEX [' + [name] COLLATE SQL_Latin1_General_CP1_CI_AS + '] ON [' +  OBJECT_NAME(object_id) + '] (' +   REVERSE(SUBSTRING(REVERSE((   SELECT name + CASE WHEN sc.is_descending_key = 1 THEN ' DESC' ELSE ' ASC' END + ','   
                FROM  sys.index_columns sc  
                JOIN sys.columns c ON sc.object_id = c.object_id AND sc.column_id = c.column_id   
                WHERE  OBJECT_NAME(sc.object_id) = @TableName AND  sc.object_id = i.object_id AND  sc.index_id = i.index_id   
                ORDER BY index_column_id ASC   FOR XML PATH('')    )), 2, 8000)) + ')'    
                FROM sys.indexes i    
                WHERE   OBJECT_NAME(object_id) = @TableName
                AND CASE WHEN @ClusteredPK = 1 AND is_primary_key = 1 AND type = 1 THEN 0 ELSE 1 END = 1   AND is_unique_constraint = 0   AND is_primary_key = 0 
               
            */
            
            INSERT INTO @MainDefinition(FieldValue)   
            SELECT FieldValue FROM @Definition    
            ORDER BY DefinitionID ASC 
            
            ----------------------------------
            --SELECT FieldValue+'' FROM @MainDefinition FOR XML PATH('')
            set @script='use '+@DBName+' '+(SELECT FieldValue+'' FROM @MainDefinition FOR XML PATH(''))
            
            --declare @q    varchar(max)
            --set @q=(select replace((SELECT FieldValue FROM @MainDefinition FOR XML PATH('')),'</FieldValue>',''))
            --set @script=(select REPLACE(@q,'<FieldValue>',''))
            --drop type TableType
END try
-- ##############################################################################################################################################################################
BEGIN CATCH        
    BEGIN
        -- INIZIO  Procedura in errore =========================================================================================================================================================
            PRINT '***********************************************************************************************************************************************************' 
            PRINT 'ErrorNumber               : ' + CAST(ERROR_NUMBER() AS NVARCHAR(MAX))
            PRINT 'ErrorSeverity             : ' + CAST(ERROR_SEVERITY() AS NVARCHAR(MAX)) 
            PRINT 'ErrorState                : ' + CAST(ERROR_STATE() AS NVARCHAR(MAX)) 
            PRINT 'ErrorLine                 : ' + CAST(ERROR_LINE() AS NVARCHAR(MAX)) 
            PRINT 'ErrorMessage              : ' + CAST(ERROR_MESSAGE() AS NVARCHAR(MAX))
            PRINT '***********************************************************************************************************************************************************' 
        -- FINE  Procedura in errore =========================================================================================================================================================
    END 
        set @script=''
    return -1
END CATCH   
-- ##############################################################################################################################################################################   

to exec it:

declare @s varchar(max)
exec [util_ScriptTable]   'db','schema_source','table_source',1,1,'schema_dest','tab_dest',0,@s output
select @s

Postgres manually alter sequence

setval('sequence_name', sequence_value)

What do 'real', 'user' and 'sys' mean in the output of time(1)?

To expand on the accepted answer, I just wanted to provide another reason why real ? user + sys.

Keep in mind that real represents actual elapsed time, while user and sys values represent CPU execution time. As a result, on a multicore system, the user and/or sys time (as well as their sum) can actually exceed the real time. For example, on a Java app I'm running for class I get this set of values:

real    1m47.363s
user    2m41.318s
sys     0m4.013s

How to vertically center a container in Bootstrap?

My prefered technique :

body {
  display: table;
  position: absolute;
  height: 100%;
  width: 100%;
}

.jumbotron {
   display: table-cell;
   vertical-align: middle;
}

Demo

_x000D_
_x000D_
body {_x000D_
  display: table;_x000D_
  position: absolute;_x000D_
  height: 100%;_x000D_
  width: 100%;_x000D_
}_x000D_
_x000D_
.jumbotron {_x000D_
   display: table-cell;_x000D_
   vertical-align: middle;_x000D_
}
_x000D_
<link rel="stylesheet" href="https://maxcdn.bootstrapcdn.com/bootstrap/3.3.6/css/bootstrap.min.css">_x000D_
<div class="jumbotron vertical-center">_x000D_
  <div class="container text-center">_x000D_
    <h1>The easiest and powerful way</h1>_x000D_
    <div class="row">_x000D_
      <div class="col-md-7">_x000D_
        <div class="top-bg">Lorem ipsum dolor sit amet, consectetur adipiscing elit, sed do eiusmod tempor incididunt ut labore et dolore magna aliqua. Ut enim ad minim veniam, quis nostrud exercitation ullamco laboris nisi ut aliquip ex ea commodo consequat. Duis aute irure dolor in reprehenderit in voluptate velit esse cillum dolore eu fugiat nulla pariatur. Excepteur sint occaecat cupidatat non proident, sunt in culpa qui officia deserunt mollit anim id est laborum.</div>_x000D_
      </div>_x000D_
_x000D_
      <div class="col-md-5 iPhone-features">_x000D_
        <ul class="top-features">_x000D_
          <li>_x000D_
            <span><i class="fa fa-random simple_bg top-features-bg"></i></span>_x000D_
            <p><strong>Redirect</strong><br>Visitors where they converts more.</p>_x000D_
          </li>_x000D_
          <li>_x000D_
            <span><i class="fa fa-cogs simple_bg top-features-bg"></i></span>_x000D_
            <p><strong>Track</strong><br>Views, Clicks and Conversions.</p>_x000D_
          </li>_x000D_
          <li>_x000D_
            <span><i class="fa fa-check simple_bg top-features-bg"></i></span>_x000D_
            <p><strong>Check</strong><br>Constantly the status of your links.</p>_x000D_
          </li>_x000D_
          <li>_x000D_
            <span><i class="fa fa-users simple_bg top-features-bg"></i></span>_x000D_
            <p><strong>Collaborate</strong><br>With Customers, Partners and Co-Workers.</p>_x000D_
          </li>_x000D_
          <a href="pricing-and-signup.html" class="btn-primary btn h2 lightBlue get-Started-btn">GET STARTED</a>_x000D_
          <h6 class="get-Started-sub-btn">FREE VERSION AVAILABLE!</h6>_x000D_
        </ul>_x000D_
      </div>_x000D_
    </div>_x000D_
  </div>_x000D_
</div>
_x000D_
_x000D_
_x000D_

See also this Fiddle!

Debugging WebSocket in Google Chrome

Chrome developer tools allows to see handshake request which stays pending during the opened connection, but you can't see traffic as far as I know. However you can sniff it for example.

Maintaining Session through Angular.js

Because the answer is no longer valid with a more stable version of angular, I am posting a newer solution.

PHP Page: session.php

if (!isset($_SESSION))
{    
    session_start();
}    

$_SESSION['variable'] = "hello world";

$sessions = array();

$sessions['variable'] = $_SESSION['variable'];

header('Content-Type: application/json');
echo json_encode($sessions);

Send back only the session variables you want in Angular not all of them don't want to expose more than what is needed.

JS All Together

var app = angular.module('StarterApp', []);
app.controller("AppCtrl", ['$rootScope', 'Session', function($rootScope, Session) {      
    Session.then(function(response){
        $rootScope.session = response;
    });
}]);

 app.factory('Session', function($http) {    
    return $http.get('/session.php').then(function(result) {       
        return result.data; 
    });
}); 
  • Do a simple get to get sessions using a factory.
  • If you want to make it post to make the page not visible when you just go to it in the browser you can, I'm just simplifying it
  • Add the factory to the controller
  • I use rootScope because it is a session variable that I use throughout all my code.

HTML

Inside your html you can reference your session

<html ng-app="StarterApp">

<body ng-controller="AppCtrl">
{{ session.variable }}
</body>

Android Studio not showing modules in project structure

As for me issue was that the first line in the build.gradle file of the OpenCV library.

It was something like this: apply plugin: 'com.android.application' This refers to the fact that the imported OpenCV is an application and not a library. It exists for OpenCV above 4.1.0. So change it to: Something like this: apply plugin: 'com.android.library'.

You might get an error in ApplicationId, comment it out in the gradle file.

Export HTML page to PDF on user click using JavaScript

This is because you define your "doc" variable outside of your click event. The first time you click the button the doc variable contains a new jsPDF object. But when you click for a second time, this variable can't be used in the same way anymore. As it is already defined and used the previous time.

change it to:

$(function () {

    var specialElementHandlers = {
        '#editor': function (element,renderer) {
            return true;
        }
    };
 $('#cmd').click(function () {
        var doc = new jsPDF();
        doc.fromHTML(
            $('#target').html(), 15, 15, 
            { 'width': 170, 'elementHandlers': specialElementHandlers }, 
            function(){ doc.save('sample-file.pdf'); }
        );

    });  
});

and it will work.

How to stop console from closing on exit?

You could open a command prompt, CD to the Debug or Release folder, and type the name of your exe. When I suggest this to people they think it is a lot of work, but here are the bare minimum clicks and keystrokes for this:

  • in Visual Studio, right click your project in Solution Explorer or the tab with the file name if you have a file in the solution open, and choose Open Containing Folder or Open in Windows Explorer
  • in the resulting Windows Explorer window, double-click your way to the folder with the exe
  • Shift-right-click in the background of the explorer window and choose Open Commmand Window here
  • type the first letter of your executable and press tab until the full name appears
  • press enter

I think that's 14 keystrokes and clicks (counting shift-right-click as two for example) which really isn't much. Once you have the command prompt, of course, running it again is just up-arrow, enter.

SSL peer shut down incorrectly in Java

please close the android studio and remove the file

.gradle

and

.idea

file form your project .Hope so it is helpful

Location:Go to Android studio projects->your project ->see both file remove (.gradle & .idea)

Selecting with complex criteria from pandas.DataFrame

Sure! Setup:

>>> import pandas as pd
>>> from random import randint
>>> df = pd.DataFrame({'A': [randint(1, 9) for x in range(10)],
                   'B': [randint(1, 9)*10 for x in range(10)],
                   'C': [randint(1, 9)*100 for x in range(10)]})
>>> df
   A   B    C
0  9  40  300
1  9  70  700
2  5  70  900
3  8  80  900
4  7  50  200
5  9  30  900
6  2  80  700
7  2  80  400
8  5  80  300
9  7  70  800

We can apply column operations and get boolean Series objects:

>>> df["B"] > 50
0    False
1     True
2     True
3     True
4    False
5    False
6     True
7     True
8     True
9     True
Name: B
>>> (df["B"] > 50) & (df["C"] == 900)
0    False
1    False
2     True
3     True
4    False
5    False
6    False
7    False
8    False
9    False

[Update, to switch to new-style .loc]:

And then we can use these to index into the object. For read access, you can chain indices:

>>> df["A"][(df["B"] > 50) & (df["C"] == 900)]
2    5
3    8
Name: A, dtype: int64

but you can get yourself into trouble because of the difference between a view and a copy doing this for write access. You can use .loc instead:

>>> df.loc[(df["B"] > 50) & (df["C"] == 900), "A"]
2    5
3    8
Name: A, dtype: int64
>>> df.loc[(df["B"] > 50) & (df["C"] == 900), "A"].values
array([5, 8], dtype=int64)
>>> df.loc[(df["B"] > 50) & (df["C"] == 900), "A"] *= 1000
>>> df
      A   B    C
0     9  40  300
1     9  70  700
2  5000  70  900
3  8000  80  900
4     7  50  200
5     9  30  900
6     2  80  700
7     2  80  400
8     5  80  300
9     7  70  800

Note that I accidentally typed == 900 and not != 900, or ~(df["C"] == 900), but I'm too lazy to fix it. Exercise for the reader. :^)

Why shouldn't I use PyPy over CPython if PyPy is 6.3 times faster?

Because pypy is not 100% compatible, takes 8 gigs of ram to compile, is a moving target, and highly experimental, where cpython is stable, the default target for module builders for 2 decades (including c extensions that don't work on pypy), and already widely deployed.

Pypy will likely never be the reference implementation, but it is a good tool to have.

Does HTML5 <video> playback support the .avi format?

Short answer: No. Use WebM or Ogg instead.

This article covers just about everything you need to know about the <video> element, including which browsers support which container formats and codecs.

Delete column from SQLite table

Instead of dropping the backup table, just rename it...

BEGIN TRANSACTION;
CREATE TABLE t1_backup(a,b);
INSERT INTO t1_backup SELECT a,b FROM t1;
DROP TABLE t1;
ALTER TABLE t1_backup RENAME TO t1;
COMMIT;

How to assert two list contain the same elements in Python?

As of Python 3.2 unittest.TestCase.assertItemsEqual(doc) has been replaced by unittest.TestCase.assertCountEqual(doc) which does exactly what you are looking for, as you can read from the python standard library documentation. The method is somewhat misleadingly named but it does exactly what you are looking for.

a and b have the same elements in the same number, regardless of their order

Here a simple example which compares two lists having the same elements but in a different order.

  • using assertCountEqual the test will succeed
  • using assertListEqual the test will fail due to the order difference of the two lists

Here a little example script.

import unittest


class TestListElements(unittest.TestCase):
    def setUp(self):
        self.expected = ['foo', 'bar', 'baz']
        self.result = ['baz', 'foo', 'bar']

    def test_count_eq(self):
        """Will succeed"""
        self.assertCountEqual(self.result, self.expected)

    def test_list_eq(self):
        """Will fail"""
        self.assertListEqual(self.result, self.expected)

if __name__ == "__main__":
    unittest.main()

Side Note : Please make sure that the elements in the lists you are comparing are sortable.

SSH to AWS Instance without key pairs

AWS added a new feature to connect to instance without any open port, the AWS SSM Session Manager. https://aws.amazon.com/blogs/aws/new-session-manager/

I've created a neat SSH ProxyCommand script that temporary adds your public ssh key to target instance during connection to target instance. The nice thing about this is you will connect without the need to add the ssh(22) port to your security groups, because the ssh connection is tunneled through ssm session manager.

AWS SSM SSH ProxyComand -> https://gist.github.com/qoomon/fcf2c85194c55aee34b78ddcaa9e83a1

How to see full query from SHOW PROCESSLIST

If one want to keep getting updated processes (on the example, 2 seconds) on a shell session without having to manually interact with it use:

watch -n 2 'mysql -h 127.0.0.1 -P 3306 -u some_user -psome_pass some_database -e "show full processlist;"'

The only bad thing about the show [full] processlist is that you can't filter the output result. On the other hand, issuing the SELECT * FROM INFORMATION_SCHEMA.PROCESSLIST open possibilities to remove from the output anything you don't want to see:

SELECT * from INFORMATION_SCHEMA.PROCESSLIST
WHERE DB = 'somedatabase'
AND COMMAND <> 'Sleep'
AND HOST NOT LIKE '10.164.25.133%' \G

Read text file into string array (and write)

You can use os.File (which implements the io.Reader interface) with the bufio package for that. However, those packages are build with fixed memory usage in mind (no matter how large the file is) and are quite fast.

Unfortunately this makes reading the whole file into the memory a bit more complicated. You can use a bytes.Buffer to join the parts of the line if they exceed the line limit. Anyway, I recommend you to try to use the line reader directly in your project (especially if do not know how large the text file is!). But if the file is small, the following example might be sufficient for you:

package main

import (
    "os"
    "bufio"
    "bytes"
    "fmt"
)

// Read a whole file into the memory and store it as array of lines
func readLines(path string) (lines []string, err os.Error) {
    var (
        file *os.File
        part []byte
        prefix bool
    )
    if file, err = os.Open(path); err != nil {
        return
    }
    reader := bufio.NewReader(file)
    buffer := bytes.NewBuffer(make([]byte, 1024))
    for {
        if part, prefix, err = reader.ReadLine(); err != nil {
            break
        }
        buffer.Write(part)
        if !prefix {
            lines = append(lines, buffer.String())
            buffer.Reset()
        }
    }
    if err == os.EOF {
        err = nil
    }
    return
}

func main() {
    lines, err := readLines("foo.txt")
    if err != nil {
        fmt.Println("Error: %s\n", err)
        return
    }
    for _, line := range lines {
        fmt.Println(line)
    }
}

Another alternative might be to use io.ioutil.ReadAll to read in the complete file at once and do the slicing by line afterwards. I don't give you an explicit example of how to write the lines back to the file, but that's basically an os.Create() followed by a loop similar to that one in the example (see main()).

ImportError: No module named 'Tkinter'

use below.

from tkinter import *
root=Tk()
.....
root.mainloop()

How to get a list of installed android applications and pick one to run

If there are multiple launchers in a one package above code has a problem. Eg: on LG Optimus Facebook for LG, MySpace for LG, Twitter for LG contains in a one package name SNS and if you use above SNS will repeat. After hours of research I came with below code. Seems to work well.

private List<String> getInstalledComponentList()
            throws NameNotFoundException {
        final Intent mainIntent = new Intent(Intent.ACTION_MAIN, null);
        mainIntent.addCategory(Intent.CATEGORY_LAUNCHER);
        List<ResolveInfo> ril = getPackageManager().queryIntentActivities(mainIntent, 0);
        List<String> componentList = new ArrayList<String>();
        String name = null;

        for (ResolveInfo ri : ril) {
            if (ri.activityInfo != null) {
                Resources res = getPackageManager().getResourcesForApplication(ri.activityInfo.applicationInfo);
                if (ri.activityInfo.labelRes != 0) {
                    name = res.getString(ri.activityInfo.labelRes);
                } else {
                    name = ri.activityInfo.applicationInfo.loadLabel(
                            getPackageManager()).toString();
                }
                componentList.add(name);
            }
        }
        return componentList;
    }

Invoking Java main method with parameters from Eclipse

I'm not sure what your uses are, but I find it convenient that usually I use no more than several command line parameters, so each of those scenarios gets one run configuration, and I just pick the one I want from the Run History.

The feature you are suggesting seems a bit of an overkill, IMO.

Run Executable from Powershell script with parameters

Try quoting the argument list:

Start-Process -FilePath "C:\Program Files\MSBuild\test.exe" -ArgumentList "/genmsi/f $MySourceDirectory\src\Deployment\Installations.xml"

You can also provide the argument list as an array (comma separated args) but using a string is usually easier.

How can I change property names when serializing with Json.net?

If you don't have access to the classes to change the properties, or don't want to always use the same rename property, renaming can also be done by creating a custom resolver.

For example, if you have a class called MyCustomObject, that has a property called LongPropertyName, you can use a custom resolver like this…

public class CustomDataContractResolver : DefaultContractResolver
{
  public static readonly CustomDataContractResolver Instance = new CustomDataContractResolver ();

  protected override JsonProperty CreateProperty(MemberInfo member, MemberSerialization memberSerialization)
  {
    var property = base.CreateProperty(member, memberSerialization);
    if (property.DeclaringType == typeof(MyCustomObject))
    {
      if (property.PropertyName.Equals("LongPropertyName", StringComparison.OrdinalIgnoreCase))
      {
        property.PropertyName = "Short";
      }
    }
    return property;
  }
}

Then call for serialization and supply the resolver:

 var result = JsonConvert.SerializeObject(myCustomObjectInstance,
                new JsonSerializerSettings { ContractResolver = CustomDataContractResolver.Instance });

And the result will be shortened to {"Short":"prop value"} instead of {"LongPropertyName":"prop value"}

More info on custom resolvers here

Can a Byte[] Array be written to a file in C#?

You can do this using System.IO.BinaryWriter which takes a Stream so:

var bw = new BinaryWriter(File.Open("path",FileMode.OpenOrCreate);
bw.Write(byteArray);

remove objects from array by object property

I assume you used splice something like this?

for (var i = 0; i < arrayOfObjects.length; i++) {
    var obj = arrayOfObjects[i];

    if (listToDelete.indexOf(obj.id) !== -1) {
        arrayOfObjects.splice(i, 1);
    }
}

All you need to do to fix the bug is decrement i for the next time around, then (and looping backwards is also an option):

for (var i = 0; i < arrayOfObjects.length; i++) {
    var obj = arrayOfObjects[i];

    if (listToDelete.indexOf(obj.id) !== -1) {
        arrayOfObjects.splice(i, 1);
        i--;
    }
}

To avoid linear-time deletions, you can write array elements you want to keep over the array:

var end = 0;

for (var i = 0; i < arrayOfObjects.length; i++) {
    var obj = arrayOfObjects[i];

    if (listToDelete.indexOf(obj.id) === -1) {
        arrayOfObjects[end++] = obj;
    }
}

arrayOfObjects.length = end;

and to avoid linear-time lookups in a modern runtime, you can use a hash set:

const setToDelete = new Set(listToDelete);
let end = 0;

for (let i = 0; i < arrayOfObjects.length; i++) {
    const obj = arrayOfObjects[i];

    if (setToDelete.has(obj.id)) {
        arrayOfObjects[end++] = obj;
    }
}

arrayOfObjects.length = end;

which can be wrapped up in a nice function:

_x000D_
_x000D_
const filterInPlace = (array, predicate) => {_x000D_
    let end = 0;_x000D_
_x000D_
    for (let i = 0; i < array.length; i++) {_x000D_
        const obj = array[i];_x000D_
_x000D_
        if (predicate(obj)) {_x000D_
            array[end++] = obj;_x000D_
        }_x000D_
    }_x000D_
_x000D_
    array.length = end;_x000D_
};_x000D_
_x000D_
const toDelete = new Set(['abc', 'efg']);_x000D_
_x000D_
const arrayOfObjects = [{id: 'abc', name: 'oh'},_x000D_
                        {id: 'efg', name: 'em'},_x000D_
                        {id: 'hij', name: 'ge'}];_x000D_
_x000D_
filterInPlace(arrayOfObjects, obj => !toDelete.has(obj.id));_x000D_
console.log(arrayOfObjects);
_x000D_
_x000D_
_x000D_

If you don’t need to do it in place, that’s Array#filter:

const toDelete = new Set(['abc', 'efg']);
const newArray = arrayOfObjects.filter(obj => !toDelete.has(obj.id));

Excel concatenation quotes

easier answer - put the stuff in quotes in different cells and then concatenate them!

B1: rcrCheck.asp
C1: =D1&B1&E1
D1: "code in quotes" and "more code in quotes"  
E1: "

it comes out perfect (can't show you because I get a stupid dialog box about code)

easy peasy!!

How to serialize an object into a string

Simple Solution,worked for me

public static byte[] serialize(Object obj) throws IOException {
    ByteArrayOutputStream out = new ByteArrayOutputStream();
    ObjectOutputStream os = new ObjectOutputStream(out);
    os.writeObject(obj);
    return out.toByteArray();
}

How many bytes does one Unicode character take?

Simply speaking Unicode is a standard which assigned one number (called code point) to all characters of the world (Its still work in progress).

Now you need to represent this code points using bytes, thats called character encoding. UTF-8, UTF-16, UTF-6 are ways of representing those characters.

UTF-8 is multibyte character encoding. Characters can have 1 to 6 bytes (some of them may be not required right now).

UTF-32 each characters have 4 bytes a characters.

UTF-16 uses 16 bits for each character and it represents only part of Unicode characters called BMP (for all practical purposes its enough). Java uses this encoding in its strings.

Using FileSystemWatcher to monitor a directory

The reason may be that watcher is declared as local variable to a method and it is garbage collected when the method finishes. You should declare it as a class member. Try the following:

FileSystemWatcher watcher;

private void watch()
{
  watcher = new FileSystemWatcher();
  watcher.Path = path;
  watcher.NotifyFilter = NotifyFilters.LastAccess | NotifyFilters.LastWrite
                         | NotifyFilters.FileName | NotifyFilters.DirectoryName;
  watcher.Filter = "*.*";
  watcher.Changed += new FileSystemEventHandler(OnChanged);
  watcher.EnableRaisingEvents = true;
}

private void OnChanged(object source, FileSystemEventArgs e)
{
  //Copies file to another directory.
}

Get int from String, also containing letters, in Java

Just go through the string, building up an int as usual, but ignore non-number characters:

int res = 0;
for (int i=0; i < str.length(); i++) {
    char c = s.charAt(i);
    if (c < '0' || c > '9') continue;
    res = res * 10 + (c - '0');
}

Concatenating string and integer in python

in python 3.6 and newer, you can format it just like this:

new_string = f'{s} {i}'
print(new_string)

or just:

print(f'{s} {i}')

Debugging Spring configuration

Yes, Spring framework logging is very detailed, You did not mention in your post, if you are already using a logging framework or not. If you are using log4j then just add spring appenders to the log4j config (i.e to log4j.xml or log4j.properties), If you are using log4j xml config you can do some thing like this

<category name="org.springframework.beans">
    <priority value="debug" />
</category>

or

<category name="org.springframework">
    <priority value="debug" />
</category>

I would advise you to test this problem in isolation using JUnit test, You can do this by using spring testing module in conjunction with Junit. If you use spring test module it will do the bulk of the work for you it loads context file based on your context config and starts container so you can just focus on testing your business logic. I have a small example here

@RunWith(SpringJUnit4ClassRunner.class)
@ContextConfiguration(locations={"classpath:springContext.xml"})
@Transactional
public class SpringDAOTest 
{
    @Autowired
    private SpringDAO dao;

    @Autowired
    private ApplicationContext appContext;

    @Test
    public void checkConfig()
    {
        AnySpringBean bean =  appContext.getBean(AnySpringBean.class);
        Assert.assertNotNull(bean);
    }
}

UPDATE

I am not advising you to change the way you load logging but try this in your dev environment, Add this snippet to your web.xml file

<context-param>
    <param-name>log4jConfigLocation</param-name>
    <param-value>/WEB-INF/log4j.xml</param-value>
</context-param>

<listener>
    <listener-class>org.springframework.web.util.Log4jConfigListener</listener-class>
</listener>

UPDATE log4j config file


I tested this on my local tomcat and it generated a lot of logging on application start up. I also want to make a correction: use debug not info as @Rayan Stewart mentioned.

<?xml version="1.0" encoding="UTF-8" ?>
<!DOCTYPE log4j:configuration SYSTEM "log4j.dtd">

<log4j:configuration xmlns:log4j="http://jakarta.apache.org/log4j/" debug="false">
    <appender name="STDOUT" class="org.apache.log4j.ConsoleAppender">
        <param name="Threshold" value="debug" />
        <layout class="org.apache.log4j.PatternLayout">
            <param name="ConversionPattern"
                value="%d{HH:mm:ss} %p [%t]:%c{3}.%M()%L - %m%n" />
        </layout>
    </appender>

    <appender name="springAppender" class="org.apache.log4j.RollingFileAppender"> 
        <param name="file" value="C:/tomcatLogs/webApp/spring-details.log" /> 
        <param name="append" value="true" /> 
        <layout class="org.apache.log4j.PatternLayout">
            <param name="ConversionPattern"
                value="%d{MM/dd/yyyy HH:mm:ss}  [%t]:%c{5}.%M()%L %m%n" />
        </layout>
    </appender>

    <category name="org.springframework">
        <priority value="debug" />
    </category>

    <category name="org.springframework.beans">
        <priority value="debug" />
    </category>

    <category name="org.springframework.security">
        <priority value="debug" />
    </category>

    <category
        name="org.springframework.beans.CachedIntrospectionResults">
        <priority value="debug" />
    </category>

    <category name="org.springframework.jdbc.core">
        <priority value="debug" />
    </category>

    <category name="org.springframework.transaction.support.TransactionSynchronizationManager">
        <priority value="debug" />
    </category>

    <root>
        <priority value="debug" />
        <appender-ref ref="springAppender" />
        <!-- <appender-ref ref="STDOUT"/>  -->
    </root>
</log4j:configuration>

How to clear APC cache entries?

apc.ini

apc.stat = "1" will force APC to stat (check) the script on each request to determine if it has been modified. If it has been modified it will recompile and cache the new version.

If this setting is off, APC will not check, which usually means that to force APC to recheck files, the web server will have to be restarted or the cache will have to be manually cleared. Note that FastCGI web server configurations may not clear the cache on restart. On a production server where the script files rarely change, a significant performance boost can be achieved by disabled stats.

Set mouse focus and move cursor to end of input using jQuery

I have found the same thing as suggested above by a few folks. If you focus() first, then push the val() into the input, the cursor will get positioned to the end of the input value in Firefox,Chrome and IE. If you push the val() into the input field first, Firefox and Chrome position the cursor at the end, but IE positions it to the front when you focus().

$('element_identifier').focus().val('some_value') 

should do the trick (it always has for me anyway).

Export table from database to csv file

rsubmit;
options missing=0;
ods listing close;
ods csv file='\\FILE_PATH_and_Name_of_report.csv';

proc sql;
SELECT *
FROM `YOUR_FINAL_TABLE_NAME';
quit;
ods csv close;

endrsubmit;

Vba macro to copy row from table if value in table meets condition

you are describing a Problem, which I would try to solve with the VLOOKUP function rather than using VBA.

You should always consider a non-vba solution first.

Here are some application examples of VLOOKUP (or SVERWEIS in German, as i know it):

http://www.youtube.com/watch?v=RCLUM0UMLXo

http://office.microsoft.com/en-us/excel-help/vlookup-HP005209335.aspx


If you have to make it as a macro, you could use VLOOKUP as an application function - a quick solution with slow performance - or you will have to make a simillar function yourself.

If it has to be the latter, then there is need for more details on your specification, regarding performance questions.

You could copy any range to an array, loop through this array and check for your value, then copy this value to any other range. This is how i would solve this as a vba-function.

This would look something like that:

Public Sub CopyFilter()

  Dim wks As Worksheet
  Dim avarTemp() As Variant
  'go through each worksheet
  For Each wks In ThisWorkbook.Worksheets
        avarTemp = wks.UsedRange
        For i = LBound(avarTemp, 1) To UBound(avarTemp, 1)
          'check in the first column in each row
          If avarTemp(i, LBound(avarTemp, 2)) = "XYZ" Then
            'copy cell
             targetWks.Cells(1, 1) = avarTemp(i, LBound(avarTemp, 2))
          End If
        Next i
  Next wks
End Sub

Ok, now i have something nice which could come in handy for myself:

Public Function FILTER(ByRef rng As Range, ByRef lngIndex As Long) As Variant
  Dim avarTemp() As Variant
  Dim avarResult() As Variant
  Dim i As Long
  avarTemp = rng

  ReDim avarResult(0)

  For i = LBound(avarTemp, 1) To UBound(avarTemp, 1)
      If avarTemp(i, 1) = "active" Then
        avarResult(UBound(avarResult)) = avarTemp(i, lngIndex)
        'expand our result array
        ReDim Preserve avarResult(UBound(avarResult) + 1)
      End If
  Next i

  FILTER = avarResult
End Function

You can use it in your Worksheet like this =FILTER(Tabelle1!A:C;2) or with =INDEX(FILTER(Tabelle1!A:C;2);3) to specify the result row. I am sure someone could extend this to include the index functionality into FILTER or knows how to return a range like object - maybe I could too, but not today ;)

Fatal error: Class 'ZipArchive' not found in

Try to write \ZIPARCHIVE instead of ZIPARCHIVE.

Remove NA values from a vector

I ran a quick benchmark comparing the two base approaches and it turns out that x[!is.na(x)] is faster than na.omit. User qwr suggested I try purrr::dicard also - this turned out to be massively slower (though I'll happily take comments on my implementation & test!)

microbenchmark::microbenchmark(
  purrr::map(airquality,function(x) {x[!is.na(x)]}), 
  purrr::map(airquality,na.omit),
  purrr::map(airquality, ~purrr::discard(.x, .p = is.na)),
  times = 1e6)

Unit: microseconds
                                                     expr    min     lq      mean median      uq       max neval cld
 purrr::map(airquality, function(x) {     x[!is.na(x)] })   66.8   75.9  130.5643   86.2  131.80  541125.5 1e+06 a  
                          purrr::map(airquality, na.omit)   95.7  107.4  185.5108  129.3  190.50  534795.5 1e+06  b 
  purrr::map(airquality, ~purrr::discard(.x, .p = is.na)) 3391.7 3648.6 5615.8965 4079.7 6486.45 1121975.4 1e+06   c

For reference, here's the original test of x[!is.na(x)] vs na.omit:

microbenchmark::microbenchmark(
    purrr::map(airquality,function(x) {x[!is.na(x)]}), 
    purrr::map(airquality,na.omit), 
    times = 1000000)


Unit: microseconds
                                              expr  min   lq      mean median    uq      max neval cld
 map(airquality, function(x) {     x[!is.na(x)] }) 53.0 56.6  86.48231   58.1  64.8 414195.2 1e+06  a 
                          map(airquality, na.omit) 85.3 90.4 134.49964   92.5 104.9 348352.8 1e+06   b

Fastest way to ping a network range and return responsive hosts?

The following (evil) code runs more than TWICE as fast as the nmap method

for i in {1..254} ;do (ping 192.168.1.$i -c 1 -w 5  >/dev/null && echo "192.168.1.$i" &) ;done

takes around 10 seconds, where the standard nmap

nmap -sP 192.168.1.1-254

takes 25 seconds...

What are the ways to sum matrix elements in MATLAB?

1)

total = 0;
for i=1:size(A,1)
  for j=1:size(A,2)
    total = total + A(i,j);
  end
end

2)

total = sum(A(:));

How do I pass multiple parameters in Objective-C?

You need to delimit each parameter name with a ":" at the very least. Technically the name is optional, but it is recommended for readability. So you could write:

- (NSMutableArray*)getBusStops:(NSString*)busStop :(NSSTimeInterval*)timeInterval;

or what you suggested:

- (NSMutableArray*)getBusStops:(NSString*)busStop forTime:(NSSTimeInterval*)timeInterval;

Cannot start GlassFish 4.1 from within Netbeans 8.0.1 Service area

I also had this problem, it is because there is an application LISTENING to 8080 port. To solve this problem I followed the below steps:

  1. Open cmd.exe then type

    netstat -aon | find ":8080" | find "LISTENING"

  2. You will see like this result

    TCP 0.0.0.0:8080 0.0.0.0:0 LISTENING 1464

  3. Copy PID "1464".

  4. Open Task Manager (Ctrl+Alt+del), go to the details tag, then find the program or service via PID that is listening to the port 8080 then STOP it or End process.

Encode URL in JavaScript?

var myOtherUrl = 
   "http://example.com/index.html?url=" + encodeURIComponent(myUrl).replace(/%20/g,'+');

Don't forget the /g flag to replace all encoded ' '

MySQL limit from descending order

yes, you can swap these 2 queries

select * from table limit 5, 5

select * from table limit 0, 5

Having services in React application

I am from Angular as well and trying out React, as of now, one recommended(?) way seems to be using High-Order Components:

A higher-order component (HOC) is an advanced technique in React for reusing component logic. HOCs are not part of the React API, per se. They are a pattern that emerges from React’s compositional nature.

Let's say you have input and textarea and like to apply the same validation logic:

const Input = (props) => (
  <input type="text"
    style={props.style}
    onChange={props.onChange} />
)
const TextArea = (props) => (
  <textarea rows="3"
    style={props.style}
    onChange={props.onChange} >
  </textarea>
)

Then write a HOC that does validate and style wrapped component:

function withValidator(WrappedComponent) {
  return class extends React.Component {
    constructor(props) {
      super(props)

      this.validateAndStyle = this.validateAndStyle.bind(this)
      this.state = {
        style: {}
      }
    }

    validateAndStyle(e) {
      const value = e.target.value
      const valid = value && value.length > 3 // shared logic here
      const style = valid ? {} : { border: '2px solid red' }
      console.log(value, valid)
      this.setState({
        style: style
      })
    }

    render() {
      return <WrappedComponent
        onChange={this.validateAndStyle}
        style={this.state.style}
        {...this.props} />
    }
  }
}

Now those HOCs share the same validating behavior:

const InputWithValidator = withValidator(Input)
const TextAreaWithValidator = withValidator(TextArea)

render((
  <div>
    <InputWithValidator />
    <TextAreaWithValidator />
  </div>
), document.getElementById('root'));

I created a simple demo.

Edit: Another demo is using props to pass an array of functions so that you can share logic composed by multiple validating functions across HOCs like:

<InputWithValidator validators={[validator1,validator2]} />
<TextAreaWithValidator validators={[validator1,validator2]} />

Edit2: React 16.8+ provides a new feature, Hook, another nice way to share logic.

const Input = (props) => {
  const inputValidation = useInputValidation()

  return (
    <input type="text"
    {...inputValidation} />
  )
}

function useInputValidation() {
  const [value, setValue] = useState('')
  const [style, setStyle] = useState({})

  function handleChange(e) {
    const value = e.target.value
    setValue(value)
    const valid = value && value.length > 3 // shared logic here
    const style = valid ? {} : { border: '2px solid red' }
    console.log(value, valid)
    setStyle(style)
  }

  return {
    value,
    style,
    onChange: handleChange
  }
}

https://stackblitz.com/edit/react-shared-validation-logic-using-hook?file=index.js

How do I obtain crash-data from my Android application?

There is this android library called Sherlock. It gives you the full report of crash along with device and application information. Whenever a crash occurs, it displays a notification in the notification bar and on clicking of the notification, it opens the crash details. You can also share crash details with others via email or other sharing options.

Installation

android {
    dataBinding {
      enabled = true
    }
}

compile('com.github.ajitsing:sherlock:1.0.0@aar') {
    transitive = true
}

Demo

enter image description here

Grouping into interval of 5 minutes within a time range

I found out that with MySQL probably the correct query is the following:

SELECT SUBSTRING( FROM_UNIXTIME( CEILING( timestamp /300 ) *300,  
                                 '%Y-%m-%d %H:%i:%S' ) , 1, 19 ) AS ts_CEILING,
SUM(value)
FROM group_interval
GROUP BY SUBSTRING( FROM_UNIXTIME( CEILING( timestamp /300 ) *300,  
                                   '%Y-%m-%d %H:%i:%S' ) , 1, 19 )
ORDER BY SUBSTRING( FROM_UNIXTIME( CEILING( timestamp /300 ) *300,  
                                   '%Y-%m-%d %H:%i:%S' ) , 1, 19 ) DESC

Let me know what you think.

#define macro for debug printing in C?

I would do something like

#ifdef DEBUG
#define debug_print(fmt, ...) fprintf(stderr, fmt, __VA_ARGS__)
#else
#define debug_print(fmt, ...) do {} while (0)
#endif

I think this is cleaner.

Best way to Format a Double value to 2 Decimal places

No, there is no better way.

Actually you have an error in your pattern. What you want is:

DecimalFormat df = new DecimalFormat("#.00"); 

Note the "00", meaning exactly two decimal places.

If you use "#.##" (# means "optional" digit), it will drop trailing zeroes - ie new DecimalFormat("#.##").format(3.0d); prints just "3", not "3.00".

How to search in a List of Java object

If you always search based on value3, you could store the objects in a Map:

Map<String, List<Sample>> map = new HashMap <>();

You can then populate the map with key = value3 and value = list of Sample objects with that same value3 property.

You can then query the map:

List<Sample> allSamplesWhereValue3IsDog = map.get("Dog");

Note: if no 2 Sample instances can have the same value3, you can simply use a Map<String, Sample>.

Help needed with Median If in Excel

one solution could be to find a way of pulling the numbers from the string and placing them in a column of just numbers the using the =MEDIAN() function giving the new number column as the range

How do you specify a different port number in SQL Management Studio?

Using the client manager affects all connections or sets a client machine specific alias.

Use the comma as above: this can be used in an app.config too

It's probably needed if you have firewalls between you and the server too...

Server.MapPath("."), Server.MapPath("~"), Server.MapPath(@"\"), Server.MapPath("/"). What is the difference?

1) Server.MapPath(".") -- Returns the "Current Physical Directory" of the file (e.g. aspx) being executed.

Ex. Suppose D:\WebApplications\Collage\Departments

2) Server.MapPath("..") -- Returns the "Parent Directory"

Ex. D:\WebApplications\Collage

3) Server.MapPath("~") -- Returns the "Physical Path to the Root of the Application"

Ex. D:\WebApplications\Collage

4) Server.MapPath("/") -- Returns the physical path to the root of the Domain Name

Ex. C:\Inetpub\wwwroot

How to create a file in Linux from terminal window?

Depending on what you want the file to contain:

  • touch /path/to/file for an empty file
  • somecommand > /path/to/file for a file containing the output of some command.

      eg: grep --help > randomtext.txt
          echo "This is some text" > randomtext.txt
    
  • nano /path/to/file or vi /path/to/file (or any other editor emacs,gedit etc)
    It either opens the existing one for editing or creates & opens the empty file to enter, if it doesn't exist


Create the file using cat

$ cat > myfile.txt

Now, just type whatever you want in the file:

Hello World!

CTRL-D to save and exit


There are several possible solutions:

Create an empty file

touch file

>file

echo -n > file

printf '' > file

The echo version will work only if your version of echo supports the -n switch to suppress newlines. This is a non-standard addition. The other examples will all work in a POSIX shell.

Create a file containing a newline and nothing else

echo '' > file

printf '\n' > file

This is a valid "text file" because it ends in a newline.

Write text into a file

"$EDITOR" file

echo 'text' > file

cat > file <<END \
text
END

printf 'text\n' > file

These are equivalent. The $EDITOR command assumes that you have an interactive text editor defined in the EDITOR environment variable and that you interactively enter equivalent text. The cat version presumes a literal newline after the \ and after each other line. Other than that these will all work in a POSIX shell.

Of course there are many other methods of writing and creating files, too.

How to force NSLocalizedString to use a specific language

You could build a sub-bundle with the set of localized strings that you want to do this with, and then use NSLocalizedStringFromTableInBundle() to load them. (I'm assuming that this is content separate from the normal UI localization you might be doing on the app.)

Sanitizing strings to make them URL and filename safe?

There are already several solutions provided for this question but I have read and tested most of the code here and I ended up with this solution which is a mix of what I learned here:

The function

The function is bundled here in a Symfony2 bundle but it can be extracted to be used as plain PHP, it only has a dependency with the iconv function that must be enabled:

Filesystem.php:

<?php

namespace COil\Bundle\COilCoreBundle\Component\HttpKernel\Util;

use Symfony\Component\HttpKernel\Util\Filesystem as BaseFilesystem;

/**
 * Extends the Symfony filesystem object.
 */
class Filesystem extends BaseFilesystem
{
    /**
     * Make a filename safe to use in any function. (Accents, spaces, special chars...)
     * The iconv function must be activated.
     *
     * @param string  $fileName       The filename to sanitize (with or without extension)
     * @param string  $defaultIfEmpty The default string returned for a non valid filename (only special chars or separators)
     * @param string  $separator      The default separator
     * @param boolean $lowerCase      Tells if the string must converted to lower case
     *
     * @author COil <https://github.com/COil>
     * @see    http://stackoverflow.com/questions/2668854/sanitizing-strings-to-make-them-url-and-filename-safe
     *
     * @return string
     */
    public function sanitizeFilename($fileName, $defaultIfEmpty = 'default', $separator = '_', $lowerCase = true)
    {
    // Gather file informations and store its extension
    $fileInfos = pathinfo($fileName);
    $fileExt   = array_key_exists('extension', $fileInfos) ? '.'. strtolower($fileInfos['extension']) : '';

    // Removes accents
    $fileName = @iconv('UTF-8', 'us-ascii//TRANSLIT', $fileInfos['filename']);

    // Removes all characters that are not separators, letters, numbers, dots or whitespaces
    $fileName = preg_replace("/[^ a-zA-Z". preg_quote($separator). "\d\.\s]/", '', $lowerCase ? strtolower($fileName) : $fileName);

    // Replaces all successive separators into a single one
    $fileName = preg_replace('!['. preg_quote($separator).'\s]+!u', $separator, $fileName);

    // Trim beginning and ending seperators
    $fileName = trim($fileName, $separator);

    // If empty use the default string
    if (empty($fileName)) {
        $fileName = $defaultIfEmpty;
    }

    return $fileName. $fileExt;
    }
}

The unit tests

What is interesting is that I have created PHPUnit tests, first to test edge cases and so you can check if it fits your needs: (If you find a bug, feel free to add a test case)

FilesystemTest.php:

<?php

namespace COil\Bundle\COilCoreBundle\Tests\Unit\Helper;

use COil\Bundle\COilCoreBundle\Component\HttpKernel\Util\Filesystem;

/**
 * Test the Filesystem custom class.
 */
class FilesystemTest extends \PHPUnit_Framework_TestCase
{
    /**
     * test sanitizeFilename()
     */
    public function testFilesystem()
    {
    $fs = new Filesystem();

    $this->assertEquals('logo_orange.gif', $fs->sanitizeFilename('--logö  _  __   ___   ora@@ñ--~gé--.gif'), '::sanitizeFilename() handles complex filename with specials chars');
    $this->assertEquals('coilstack', $fs->sanitizeFilename('cOiLsTaCk'), '::sanitizeFilename() converts all characters to lower case');
    $this->assertEquals('cOiLsTaCk', $fs->sanitizeFilename('cOiLsTaCk', 'default', '_', false), '::sanitizeFilename() lower case can be desactivated, passing false as the 4th argument');
    $this->assertEquals('coil_stack', $fs->sanitizeFilename('coil stack'), '::sanitizeFilename() convert a white space to a separator');
    $this->assertEquals('coil-stack', $fs->sanitizeFilename('coil stack', 'default', '-'), '::sanitizeFilename() can use a different separator as the 3rd argument');
    $this->assertEquals('coil_stack', $fs->sanitizeFilename('coil          stack'), '::sanitizeFilename() removes successive white spaces to a single separator');
    $this->assertEquals('coil_stack', $fs->sanitizeFilename('       coil stack'), '::sanitizeFilename() removes spaces at the beginning of the string');
    $this->assertEquals('coil_stack', $fs->sanitizeFilename('coil   stack         '), '::sanitizeFilename() removes spaces at the end of the string');
    $this->assertEquals('coilstack', $fs->sanitizeFilename('coil,,,,,,stack'), '::sanitizeFilename() removes non-ASCII characters');
    $this->assertEquals('coil_stack', $fs->sanitizeFilename('coil_stack  '), '::sanitizeFilename() keeps separators');
    $this->assertEquals('coil_stack', $fs->sanitizeFilename(' coil________stack'), '::sanitizeFilename() converts successive separators into a single one');
    $this->assertEquals('coil_stack.gif', $fs->sanitizeFilename('cOil Stack.GiF'), '::sanitizeFilename() lower case filename and extension');
    $this->assertEquals('copy_of_coil.stack.exe', $fs->sanitizeFilename('Copy of coil.stack.exe'), '::sanitizeFilename() keeps dots before the extension');
    $this->assertEquals('default.doc', $fs->sanitizeFilename('____________.doc'), '::sanitizeFilename() returns a default file name if filename only contains special chars');
    $this->assertEquals('default.docx', $fs->sanitizeFilename('     ___ -  --_     __%%%%__¨¨¨***____      .docx'), '::sanitizeFilename() returns a default file name if filename only contains special chars');
    $this->assertEquals('logo_edition_1314352521.jpg', $fs->sanitizeFilename('logo_edition_1314352521.jpg'), '::sanitizeFilename() returns the filename untouched if it does not need to be modified');
    $userId = rand(1, 10);
    $this->assertEquals('user_doc_'. $userId. '.doc', $fs->sanitizeFilename('?????.doc', 'user_doc_'. $userId), '::sanitizeFilename() returns the default string (the 2nd argument) if it can\'t be sanitized');
    }
}

The test results: (checked on Ubuntu with PHP 5.3.2 and MacOsX with PHP 5.3.17:

All tests pass:

phpunit -c app/ src/COil/Bundle/COilCoreBundle/Tests/Unit/Helper/FilesystemTest.php
PHPUnit 3.6.10 by Sebastian Bergmann.

Configuration read from /var/www/strangebuzz.com/app/phpunit.xml.dist

.

Time: 0 seconds, Memory: 5.75Mb

OK (1 test, 17 assertions)

Apply style to only first level of td tags

Just make a selector for tables inside a MyClass.

.MyClass td {border: solid 1px red;}
.MyClass table td {border: none}

(To generically apply to all inner tables, you could also do table table td.)

Install pip in docker

An alternative is to use the Alpine Linux containers, e.g. python:2.7-alpine. They offer pip out of the box (and have a smaller footprint which leads to faster builds etc).

Finding the number of days between two dates

TL;DR do not use UNIX timestamps. Do not use time(). If you do, be prepared should its 98.0825% reliability fail you. Use DateTime (or Carbon).

The correct answer is the one given by Saksham Gupta (other answers are also correct):

$date1 = new DateTime('2010-07-06');
$date2 = new DateTime('2010-07-09');
$days  = $date2->diff($date1)->format('%a');

Or procedurally as a one-liner:

/**
 * Number of days between two dates.
 *
 * @param date $dt1    First date
 * @param date $dt2    Second date
 * @return int
 */
function daysBetween($dt1, $dt2) {
    return date_diff(
        date_create($dt2),  
        date_create($dt1)
    )->format('%a');
}

With a caveat: the '%a' seems to indicate the absolute number of days. If you want it as a signed integer, i.e. negative when the second date is before the first, then you need to use the '%r' prefix (i.e. format('%r%a')).


If you really must use UNIX timestamps, set the time zone to GMT to avoid most of the pitfalls detailed below.


Long answer: why dividing by 24*60*60 (aka 86400) is unsafe

Most of the answers using UNIX timestamps (and 86400 to convert that to days) make two assumptions that, put together, can lead to scenarios with wrong results and subtle bugs that may be difficult to track, and arise even days, weeks or months after a successful deployment. It's not that the solution doesn't work - it works. Today. But it might stop working tomorrow.

First mistake is not considering that when asked, "How many days passed since yesterday?", a computer might truthfully answer zero if between the present and the instant indicated by "yesterday" less than one whole day has passed.

Usually when converting a "day" to a UNIX timestamp, what is obtained is the timestamp for the midnight of that particular day.

So between the midnights of October 1st and October 15th, fifteen days have elapsed. But between 13:00 of October 1st and 14:55 of October 15th, fifteen days minus 5 minutes have elapsed, and most solutions using floor() or doing implicit integer conversion will report one day less than expected.

So, "how many days ago was Y-m-d H:i:s"? will yield the wrong answer.

The second mistake is equating one day to 86400 seconds. This is almost always true - it happens often enough to overlook the times it doesn't. But the distance in seconds between two consecutive midnights is surely not 86400 at least twice a year when daylight saving time comes into play. Comparing two dates across a DST boundary will yield the wrong answer.

So even if you use the "hack" of forcing all date timestamps to a fixed hour, say midnight (this is also done implicitly by various languages and frameworks when you only specify day-month-year and not also hour-minute-second; same happens with DATE type in databases such as MySQL), the widely used formula

 FLOOR((unix_timestamp(DATE2) - unix_timestamp(DATE1)) / 86400)

or

 floor((time() - strtotime($somedate)) / 86400)

will return, say, 17 when DATE1 and DATE2 are in the same DST segment of the year; but even if the hour:minute:second part is identical, the argument might be 17.042, and worse still, 16.958 when they are in different DST segments and the time zone is DST-aware. The use of floor() or any implicit truncation to integer will then convert what should have been a 17 to a 16. In other circumstances, expressions like "$days > 17" will return true for 17.042, even if this will look as if the elapsed day count is 18.

And things grow even uglier since such code is not portable across platforms, because some of them may apply leap seconds and some might not. On those platforms that do, the difference between two dates will not be 86400 but 86401, or maybe 86399. So code that worked in May and actually passed all tests will break next June when 12.99999 days are considered 12 days instead of 13. Two dates that worked in 2015 will not work in 2017 -- the same dates, and neither year is a leap year. And between 2018-03-01 and 2017-03-01, on those platforms that care, 366 days will have passed instead of 365, making 2018 a leap year (which it is not).

So if you really want to use UNIX timestamps:

  • use round() function wisely, not floor().

  • as an alternative, do not calculate differences between D1-M1-YYY1 and D2-M2-YYY2. Those dates will be really considered as D1-M1-YYY1 00:00:00 and D2-M2-YYY2 00:00:00. Rather, convert between D1-M1-YYY1 22:30:00 and D2-M2-YYY2 04:30:00. You will always get a remainder of about twenty hours. This may become twenty-one hours or nineteen, and maybe eighteen hours, fifty-nine minutes thirty-six seconds. No matter. It is a large margin which will stay there and stay positive for the foreseeable future. Now you can truncate it with floor() in safety.

The correct solution though, to avoid magic constants, rounding kludges and a maintenance debt, is to

  • use a time library (Datetime, Carbon, whatever); don't roll your own

  • write comprehensive test cases using really evil date choices - across DST boundaries, across leap years, across leap seconds, and so on, as well as commonplace dates. Ideally (calls to datetime are fast!) generate four whole years' (and one day) worth of dates by assembling them from strings, sequentially, and ensure that the difference between the first day and the day being tested increases steadily by one. This will ensure that if anything changes in the low-level routines and leap seconds fixes try to wreak havoc, at least you will know.

  • run those tests regularly together with the rest of the test suite. They're a matter of milliseconds, and may save you literally hours of head scratching.


Whatever your solution, test it!

The function funcdiff below implements one of the solutions (as it happens, the accepted one) in a real world scenario.

<?php
$tz         = 'Europe/Rome';
$yearFrom   = 1980;
$yearTo     = 2020;
$verbose    = false;

function funcdiff($date2, $date1) {
    $now        = strtotime($date2);
    $your_date  = strtotime($date1);
    $datediff   = $now - $your_date;
    return floor($datediff / (60 * 60 * 24));
}
########################################

date_default_timezone_set($tz);
$failures   = 0;
$tests      = 0;

$dom = array ( 0, 31, 28, 31, 30,
                  31, 30, 31, 31,
                  30, 31, 30, 31 );
(array_sum($dom) === 365) || die("Thirty days hath September...");
$last   = array();
for ($year = $yearFrom; $year < $yearTo; $year++) {
    $dom[2] = 28;
    // Apply leap year rules.
    if ($year % 4 === 0)   { $dom[2] = 29; }
    if ($year % 100 === 0) { $dom[2] = 28; }
    if ($year % 400 === 0) { $dom[2] = 29; }
    for ($month = 1; $month <= 12; $month ++) {
        for ($day = 1; $day <= $dom[$month]; $day++) {
            $date = sprintf("%04d-%02d-%02d", $year, $month, $day);
            if (count($last) === 7) {
                $tests ++;
                $diff = funcdiff($date, $test = array_shift($last));
                if ((double)$diff !== (double)7) {
                    $failures ++;
                    if ($verbose) {
                        print "There seem to be {$diff} days between {$date} and {$test}\n";
                    }
                }
            }
            $last[] = $date;
        }
    }
}

print "This function failed {$failures} of its {$tests} tests";
print " between {$yearFrom} and {$yearTo}.\n";

The result is,

This function failed 280 of its 14603 tests

Horror Story: the cost of "saving time"

This actually happened some months ago. An ingenious programmer decided to save several microseconds off a calculation that took about thirty seconds at most, by plugging in the infamous "(MidnightOfDateB-MidnightOfDateA)/86400" code in several places. It was so obvious an optimization that he did not even document it, and the optimization passed the integration tests and lurked in the code for several months, all unnoticed.

This happened in a program that calculates the wages for several top-selling salesmen, the least of which has a frightful lot more clout than a whole humble five-people programmer team taken together. One day some months ago, for reasons that matter little, the bug struck -- and some of those guys got shortchanged one whole day of fat commissions. They were definitely not amused.

Infinitely worse, they lost the (already very little) faith they had in the program not being designed to surreptitiously shaft them, and pretended - and obtained - a complete, detailed code review with test cases ran and commented in layman's terms (plus a lot of red-carpet treatment in the following weeks).

What can I say: on the plus side, we got rid of a lot of technical debt, and were able to rewrite and refactor several pieces of a spaghetti mess that hearkened back to a COBOL infestation in the swinging '90s. The program undoubtedly runs better now, and there's a lot more debugging information to quickly zero in when anything looks fishy. I estimate that just this last one thing will save perhaps one or two man-days per month for the foreseeable future.

On the minus side, the whole brouhaha costed the company about €200,000 up front - plus face, plus undoubtedly some bargaining power (and, hence, yet more money).

The guy responsible for the "optimization" had changed job a year ago, before the disaster, but still there was talk to sue him for damages. And it didn't go well with the upper echelons that it was "the last guy's fault" - it looked like a set-up for us to come up clean of the matter, and in the end, we're still in the doghouse and one of the team is planning to quit.

Ninety-nine times out of one hundred, the "86400 hack" will work flawlessly. (For example in PHP, strtotime() will ignore DST, and report that between the midnights of the last Saturday of October and that of the following Monday, exactly 2 * 24 * 60 * 60 seconds have passed, even if that is plainly not true... and two wrongs will happily make one right).

This, ladies and gentlemen, was one instance when it did not. As with air-bags and seat belts, you will perhaps never really need the complexity (and ease of use) of DateTime or Carbon. But the day when you might (or the day when you'll have to prove you thought about this) will come as a thief in the night. Be prepared.

How to make html table vertically scrollable

Just add the display:block to the thead > tr and tbody. check the below example

http://www.imaputz.com/cssStuff/bigFourVersion.html

How can I create an object based on an interface file definition in TypeScript?

You can set default values using Class.

Without Class Constructor:

interface IModal {
  content: string;
  form: string;
  href: string;
  isPopup: boolean;
};

class Modal implements IModal {
  content = "";
  form = "";
  href: string;  // will not be added to object
  isPopup = true;
}

const myModal = new Modal();
console.log(myModal); // output: {content: "", form: "", isPopup: true}

With Class Constructor

interface IModal {
  content: string;
  form: string;
  href: string;
  isPopup: boolean;
}

class Modal implements IModal {
  constructor() {
    this.content = "";
    this.form = "";
    this.isPopup = true;
  }

  content: string;

  form: string;

  href: string; // not part of constructor so will not be added to object

  isPopup: boolean;
}

const myModal = new Modal();
console.log(myModal); // output: {content: "", form: "", isPopup: true}

How can I use regex to get all the characters after a specific character, e.g. comma (",")

This should work

preg_match_all('@.*\,(.*)@', '{{your data}}', $arr, PREG_PATTERN_ORDER);

You can test it here: http://www.spaweditor.com/scripts/regex/index.php

RegEx: .*\,(.*)

Same RegEx test here for JavaScript: http://www.regular-expressions.info/javascriptexample.html

What is JSONP, and why was it created?

Because you can ask the server to prepend a prefix to the returned JSON object. E.g

function_prefix(json_object);

in order for the browser to eval "inline" the JSON string as an expression. This trick makes it possible for the server to "inject" javascript code directly in the Client browser and this with bypassing the "same origin" restrictions.

In other words, you can achieve cross-domain data exchange.


Normally, XMLHttpRequest doesn't permit cross-domain data-exchange directly (one needs to go through a server in the same domain) whereas:

<script src="some_other_domain/some_data.js&prefix=function_prefix>` one can access data from a domain different than from the origin.


Also worth noting: even though the server should be considered as "trusted" before attempting that sort of "trick", the side-effects of possible change in object format etc. can be contained. If a function_prefix (i.e. a proper js function) is used to receive the JSON object, the said function can perform checks before accepting/further processing the returned data.

Run CRON job everyday at specific time

From cron manual http://man7.org/linux/man-pages/man5/crontab.5.html:

Lists are allowed. A list is a set of numbers (or ranges) separated by commas. Examples: "1,2,5,9", "0-4,8-12".

So in this case it would be:

30 10,14 * * *

Anaconda site-packages

I encountered this issue in my conda environment. The reason is that packages have been installed into two different folders, only one of which is recognised by the Python executable.

~/anaconda2/envs/[my_env]/site-packages ~/anaconda2/envs/[my_env]/lib/python2.7/site-packages

A proved solution is to add both folders to python path, using the following steps in command line (Please replace [my_env] with your own environment):

  1. conda activate [my_env].
  2. conda-develop ~/anaconda2/envs/[my_env]/site-packages
  3. conda-develop ~/anaconda2/envs/[my_env]/lib/python2.7/site-packages (conda-develop is to add a .pth file to the folder so that the Python executable knows of this folder when searching for packages.)

To ensure this works, try to activate Python in this environment, and import the package that was not found.

Spring configure @ResponseBody JSON format

AngerClown pointed me to the right direction.

This is what I finally did, just in case anyone find it useful.

<bean
    class="org.springframework.web.servlet.mvc.annotation.AnnotationMethodHandlerAdapter">
    <property name="messageConverters">
        <list>
            <bean class="org.springframework.http.converter.json.MappingJacksonHttpMessageConverter">
                <property name="objectMapper" ref="jacksonObjectMapper" />
            </bean>
        </list>
    </property>
</bean>

<!-- jackson configuration : https://stackoverflow.com/questions/3661769 -->
<bean id="jacksonObjectMapper" class="org.codehaus.jackson.map.ObjectMapper" />
<bean id="jacksonSerializationConfig" class="org.codehaus.jackson.map.SerializationConfig"
    factory-bean="jacksonObjectMapper" factory-method="getSerializationConfig" />
<bean
    class="org.springframework.beans.factory.config.MethodInvokingFactoryBean">
    <property name="targetObject" ref="jacksonSerializationConfig" />
    <property name="targetMethod" value="setSerializationInclusion" />
    <property name="arguments">
        <list>
            <value type="org.codehaus.jackson.map.annotate.JsonSerialize.Inclusion">NON_DEFAULT</value>
        </list>
    </property>
</bean>

I still have to figure out how to configure the other properties such as:

om.configure(JsonGenerator.Feature.QUOTE_FIELD_NAMES, true);

send Content-Type: application/json post with node.js

Mikeal's request module can do this easily:

var request = require('request');

var options = {
  uri: 'https://www.googleapis.com/urlshortener/v1/url',
  method: 'POST',
  json: {
    "longUrl": "http://www.google.com/"
  }
};

request(options, function (error, response, body) {
  if (!error && response.statusCode == 200) {
    console.log(body.id) // Print the shortened url.
  }
});

drop down list value in asp.net

To add items to drop down list with value in asp.net using c# just add below codes for example:

try
{
    SqlConnection conn = new SqlConnection(conStr);
    SqlCommand comm = conn.CreateCommand();
    comm = conn.CreateCommand();
    comm.CommandText = "SELECT title,gid from Groups";
    comm.CommandType = CommandType.Text;
    conn.Open();
    SqlDataReader dr = comm.ExecuteReader();
    while (dr.Read())
    {
       dropDownList.Items.Add(new 
       ListItem(dr[0].ToString(),dr[1].ToString()));
    }
}
catch (Exception e)
{
    lblText.Text = e.Message;
}
finally
{
  conn.Close();
}

What does {0} mean when found in a string in C#?

You are printing a formatted string. The {0} means to insert the first parameter following the format string; in this case the value associated with the key "rtf".

For String.Format, which is similar, if you had something like

//            Format string                    {0}           {1}
String.Format("This {0}.  The value is {1}.",  "is a test",  42 ) 

you'd create a string "This is a test. The value is 42".

You can also use expressions, and print values out multiple times:

//            Format string              {0} {1}  {2}
String.Format("Fib: {0}, {0}, {1}, {2}", 1,  1+1, 1+2) 

yielding "Fib: 1, 1, 2, 3"

See more at http://msdn.microsoft.com/en-us/library/txafckwd.aspx, which talks about composite formatting.

UITableView - change section header color

I have a project using static table view cells, in iOS 7.x. willDisplayHeaderView does not fire. However, this method works ok:

- (UIView *)tableView:(UITableView *)tableView viewForHeaderInSection:(NSInteger)section
{
    NSLog(@"%s", __FUNCTION__);
    CGRect headerFrame = CGRectMake(x, y, w, h);    
    UIView *headerView = [[UIView alloc] initWithFrame:headerFrame];  
    headerView.backgroundColor = [UIColor blackColor];

git cherry-pick says "...38c74d is a merge but no -m option was given"

Simplification of @Daira Hopwood method good for picking one single commit. Need no temporary branches.

In the case of the author:

  • Z is wanted commit (fd9f578)
  • Y is commit before it
  • X current working branch

then do:

git checkout Z   # move HEAD to wanted commit
git reset Y      # have Z as changes in working tree
git stash        # save Z in stash
git checkout X   # return to working branch
git stash pop    # apply Z to current branch
git commit -a    # do commit

Wrap text in <td> tag

table-layout:fixed will resolve the expanding cell problem, but will create a new one. IE by default will hide the overflow but Mozilla will render it outside the box.

Another solution would be to use: overflow:hidden;width:?px

<table style="table-layout:fixed; width:100px">
 <tr>
   <td style="overflow:hidden; width:50px;">fearofthedarkihaveaconstantfearofadark</td>
   <td>
     test
   </td>
 </tr>
</table>

Can't compile C program on a Mac after upgrade to Mojave

After trying every answer I could find here and online, I was still getting errors for some missing headers. When trying to compile pyRFR, I was getting errors about stdexcept not being found, which apparently was not installed in /usr/include with the other headers. However, I found where it was hiding in Mojave and added this to the end of my ~/.bash_profile file:

export CPATH=/Library/Developer/CommandLineTools/usr/include/c++/v1

Having done that, I can now compile pyRFR and other C/C++ programs. According to echo | gcc -E -Wp,-v -, gcc was looking in the old location for these headers (without the /c++/v1), but not the new location, so adding that to CFLAGS fixed it.

SQL query to make all data in a column UPPER CASE?

Permanent:

UPDATE
  MyTable
SET
  MyColumn = UPPER(MyColumn)

Temporary:

SELECT
  UPPER(MyColumn) AS MyColumn
FROM
  MyTable

swift 3.0 Data to String?

Swift 4 version of 4redwings's answer:

let testString = "This is a test string"
let somedata = testString.data(using: String.Encoding.utf8)
let backToString = String(data: somedata!, encoding: String.Encoding.utf8)

Django TemplateDoesNotExist?

If you encounter this problem when you add an app from scratch. It is probably because that you miss some settings. Three steps is needed when adding an app.

1?Create the directory and template file.

Suppose you have a project named mysite and you want to add an app named your_app_name. Put your template file under mysite/your_app_name/templates/your_app_name as following.

+-- mysite
¦   +-- settings.py
¦   +-- urls.py
¦   +-- wsgi.py
+-- your_app_name
¦   +-- admin.py
¦   +-- apps.py
¦   +-- models.py
¦   +-- templates
¦   ¦   +-- your_app_name
¦   ¦       +-- my_index.html
¦   +-- urls.py
¦   +-- views.py

2?Add your app to INSTALLED_APPS.

Modify settings.py

INSTALLED_APPS = [
    ...
    'your_app_name',
    ...
]

3?Add your app directory to DIRS in TEMPLATES.

Modify settings.py.

TEMPLATES = [
    {
        ...
        'DIRS': [os.path.join(BASE_DIR, 'templates'),
                 os.path.join(BASE_DIR, 'your_app_name', 'templates', 'your_app_name'),
                ...
                ]
    }
]

Stretch child div height to fill parent that has dynamic height

You can do it easily with a bit of jQuery

$(document).ready(function(){
  var parentHeight = $("#parentDiv").parent().height();
  $("#childDiv").height(parentHeight);
});

Permission denied error on Github Push

For some reason my push and pull origin was changed to HTTPS-url in stead of SSH-url (probably a copy-paste error on my end), but trying to push would give me the following error after trying to login:

Username for 'https://github.com': xxx
Password for 'https://[email protected]': 
remote: Invalid username or password.

Updating the remote origin with the SSH url, solved the problem:

git remote set-url origin [email protected]:<username>/<repo>.git

Hope this helps!

How to set proper codeigniter base url?

Check

config > config

codeigniter file structure

replace

$config['base_url'] = "your Website url";

with

$config['base_url']  =  "http://".$_SERVER['HTTP_HOST'];

$config['base_url'] .= preg_replace('@/+$@', '', dirname($_SERVER['SCRIPT_NAME'])).'/';

Bootstrap: Use .pull-right without having to hardcode a negative margin-top

Keep the h2 at the top, then pull-left on the p and pull-right on the login-box

<div class='container'>
  <div class='hero-unit'>

    <h2>Welcome</h2>

    <div class="pull-left">
      <p>Please log in</p>
    </div>

    <div id='login-box' class='pull-right control-group'>
        <div class='clearfix'>
            <input type='text' placeholder='Username' />
        </div>
        <div class='clearfix'>
            <input type='password' placeholder='Password' />
        </div>
        <button type='button' class='btn btn-primary'>Log in</button>
    </div>

    <div class="clearfix"></div>

  </div>
</div>

the default vertical-align on floated boxes is baseline, so the "Please log in" exactly lines up with the "Username" (check by changing the pull-right to pull-left).

How can I use jQuery to make an input readonly?

<html xmlns="http://www.w3.org/1999/xhtml">
<head >
    <title></title>

    <script type="text/javascript" src="http://ajax.googleapis.com/ajax/libs/jquery/1.3.2/jquery.min.js"></script>

</head>
<body>
    <div>
        <input id="fieldName" name="fieldName" type="text" class="text_box" value="Firstname" />
    </div>
</body>

<script type="text/javascript">
    $(function()
    {
        $('#fieldName').attr('disabled', 'disabled');

    });
</script>
</html>

Create a new workspace in Eclipse

You can create multiple workspaces in Eclipse. You have to just specify the path of the workspace during Eclipse startup. You can even switch workspaces via File?Switch workspace.

You can then import project to your workspace, copy paste project to your new workspace folder, then

File?Import?Existing project in to workspace?select project.

How to override toString() properly in Java?

The toString is supposed to return a String.

public String toString() { 
    return "Name: '" + this.name + "', Height: '" + this.height + "', Birthday: '" + this.bDay + "'";
} 

I suggest you make use of your IDE's features to generate the toString method. Don't hand-code it.

For instance, Eclipse can do so if you simply right-click on the source code and select Source > Generate toString

Execute jar file with multiple classpath libraries from command prompt

There are several options.

The easiest is likely the exec plugin.

You can also generate a jar containing all the dependencies using the assembly plugin.

Lastly, you can generate a file with the classpath in it using the dependency:classpath goal.

LIMIT 10..20 in SQL Server

select * from (select id,name,ROW_NUMBER() OVER (ORDER BY id  asc) as row
from tableName1) tbl1
where tbl1.row>=10 and tbl1.row<=15

Will print rows from 10 to 15.

How do I add options to a DropDownList using jQuery?

And also, use .prepend() to add the option to the start of the options list. http://api.jquery.com/prepend/

MongoDB relationships: embed or reference?

This is more an art than a science. The Mongo Documentation on Schemas is a good reference, but here are some things to consider:

  • Put as much in as possible

    The joy of a Document database is that it eliminates lots of Joins. Your first instinct should be to place as much in a single document as you can. Because MongoDB documents have structure, and because you can efficiently query within that structure (this means that you can take the part of the document that you need, so document size shouldn't worry you much) there is no immediate need to normalize data like you would in SQL. In particular any data that is not useful apart from its parent document should be part of the same document.

  • Separate data that can be referred to from multiple places into its own collection.

    This is not so much a "storage space" issue as it is a "data consistency" issue. If many records will refer to the same data it is more efficient and less error prone to update a single record and keep references to it in other places.

  • Document size considerations

    MongoDB imposes a 4MB (16MB with 1.8) size limit on a single document. In a world of GB of data this sounds small, but it is also 30 thousand tweets or 250 typical Stack Overflow answers or 20 flicker photos. On the other hand, this is far more information than one might want to present at one time on a typical web page. First consider what will make your queries easier. In many cases concern about document sizes will be premature optimization.

  • Complex data structures:

    MongoDB can store arbitrary deep nested data structures, but cannot search them efficiently. If your data forms a tree, forest or graph, you effectively need to store each node and its edges in a separate document. (Note that there are data stores specifically designed for this type of data that one should consider as well)

    It has also been pointed out than it is impossible to return a subset of elements in a document. If you need to pick-and-choose a few bits of each document, it will be easier to separate them out.

  • Data Consistency

    MongoDB makes a trade off between efficiency and consistency. The rule is changes to a single document are always atomic, while updates to multiple documents should never be assumed to be atomic. There is also no way to "lock" a record on the server (you can build this into the client's logic using for example a "lock" field). When you design your schema consider how you will keep your data consistent. Generally, the more that you keep in a document the better.

For what you are describing, I would embed the comments, and give each comment an id field with an ObjectID. The ObjectID has a time stamp embedded in it so you can use that instead of created at if you like.

How to insert a value that contains an apostrophe (single quote)?

You just have to double up on the single quotes...

insert into Person (First, Last)
values ('Joe', 'O''Brien')

MySQL: Error dropping database (errno 13; errno 17; errno 39)

In my case it was due to 'lower_case_table_names' parameter.

The error number 39 thrown out when I tried to drop the databases which consists upper case table names with lower_case_table_names parameter is enabled.

This is fixed by reverting back the lower case parameter changes to the previous state.

how to get current datetime in SQL?

I want my datetime, and I want it now()!

For MySQL, anyway.

How to INNER JOIN 3 tables using CodeIgniter

it should be like that,

$this->db->select('*');    
$this->db->from('table1');
$this->db->join('table2', 'table1.id = table2.id');
$this->db->join('table3', 'table1.id = table3.id');
$query = $this->db->get();

as per CodeIgniters active record framework

Allow only numbers and dot in script

Instead of using this:

onkeypress="return fun_AllowOnlyAmountAndDot(this);"

You should use this:

onkeypress="return fun_AllowOnlyAmountAndDot(this.id);"

How to parse JSON with VBA without external libraries?

There are two issues here. The first is to access fields in the array returned by your JSON parse, the second is to rename collections/fields (like sentences) away from VBA reserved names.

Let's address the second concern first. You were on the right track. First, replace all instances of sentences with jsentences If text within your JSON also contains the word sentences, then figure out a way to make the replacement unique, such as using "sentences":[ as the search string. You can use the VBA Replace method to do this.

Once that's done, so VBA will stop renaming sentences to Sentences, it's just a matter of accessing the array like so:

'first, declare the variables you need:
Dim jsent as Variant

'Get arr all setup, then
For Each jsent in arr.jsentences
  MsgBox(jsent.orig)
Next

Executing a command stored in a variable from PowerShell

Here is yet another way without Invoke-Expression but with two variables (command:string and parameters:array). It works fine for me. Assume 7z.exe is in the system path.

$cmd = '7z.exe'
$prm = 'a', '-tzip', 'c:\temp\with space\test1.zip', 'C:\TEMP\with space\changelog'

& $cmd $prm

If the command is known (7z.exe) and only parameters are variable then this will do

$prm = 'a', '-tzip', 'c:\temp\with space\test1.zip', 'C:\TEMP\with space\changelog'

& 7z.exe $prm

BTW, Invoke-Expression with one parameter works for me, too, e.g. this works

$cmd = '& 7z.exe a -tzip "c:\temp\with space\test2.zip" "C:\TEMP\with space\changelog"'

Invoke-Expression $cmd

P.S. I usually prefer the way with a parameter array because it is easier to compose programmatically than to build an expression for Invoke-Expression.

Create a copy of a table within the same database DB2

We can copy all columns from one table to another, existing table:

INSERT INTO table2 SELECT * FROM table1;

Or we can copy only the columns we want to into another, existing table:

INSERT INTO table2 (column_name(s)) SELECT column_name(s) FROM table1;

or SELECT * INTO BACKUP_TABLE1 FROM TABLE1

Hidden Features of Xcode

The User Scripts menu has a lot of goodies in it, and it's relatively easy to add your own. For example, I added a shortcut and bound it to cmd-opt-- to insert a comment divider and a #pragma mark in my code to quickly break up a file.

  #!/bin/sh
  echo -n "//================....================
  #pragma mark "

When I hit cmd-opt--, these lines are inserted into my code and the cursor is pre-positioned to edit the pragma mark component, which shows up in the symbol popup.

What is define([ , function ]) in JavaScript?

That's probably a requireJS module definition

Check here for more details

RequireJS is a JavaScript file and module loader. It is optimized for in-browser use, but it can be used in other JavaScript environments, like Rhino and Node. Using a modular script loader like RequireJS will improve the speed and quality of your code.

How to set value of input text using jQuery

Here is another variation for a file upload that has a nicer looking bootstrap button than the default file upload browse button. This is the html:

<div class="form-group">
        @Html.LabelFor(model => model.FileName, htmlAttributes: new { @class = "col-md-2  control-label" })
        <div class="col-md-1 btn btn-sn btn-primary" id="browseButton" onclick="$(this).parent().find('input[type=file]').click();">browse</div>
        <div class="col-md-7">
            <input id="fileSpace" name="uploaded_file"  type="file" style="display: none;">   @*style="display: none;"*@
            @Html.EditorFor(model => model.FileName, new { htmlAttributes = new { @class = "form-control", @id = "modelField"} })
            @Html.ValidationMessageFor(model => model.FileName, "", new { @class = "text-danger" })
        </div>
    </div>

Here is the script:

        $('#fileSpace').on("change", function () {
        $("#modelField").val($('input[name="uploaded_file"]').val());

How do I use the Simple HTTP client in Android?

You can use like this:

public static String executeHttpPost1(String url,
            HashMap<String, String> postParameters) throws UnsupportedEncodingException {
        // TODO Auto-generated method stub

        HttpClient client = getNewHttpClient();

        try{
        request = new HttpPost(url);

        }
        catch(Exception e){
            e.printStackTrace();
        }


        if(postParameters!=null && postParameters.isEmpty()==false){

            List<NameValuePair> nameValuePairs = new ArrayList<NameValuePair>(postParameters.size());
            String k, v;
            Iterator<String> itKeys = postParameters.keySet().iterator();
            while (itKeys.hasNext()) 
            {
                k = itKeys.next();
                v = postParameters.get(k);
                nameValuePairs.add(new BasicNameValuePair(k, v));
            }     

            UrlEncodedFormEntity urlEntity  = new  UrlEncodedFormEntity(nameValuePairs);
            request.setEntity(urlEntity);

        }
        try {


            Response = client.execute(request,localContext);
            HttpEntity entity = Response.getEntity();
            int statusCode = Response.getStatusLine().getStatusCode();
            Log.i(TAG, ""+statusCode);


            Log.i(TAG, "------------------------------------------------");





                try{
                    InputStream in = (InputStream) entity.getContent(); 
                    //Header contentEncoding = Response.getFirstHeader("Content-Encoding");
                    /*if (contentEncoding != null && contentEncoding.getValue().equalsIgnoreCase("gzip")) {
                        in = new GZIPInputStream(in);
                    }*/
                    BufferedReader reader = new BufferedReader(new InputStreamReader(in));
                    StringBuilder str = new StringBuilder();
                    String line = null;
                    while((line = reader.readLine()) != null){
                        str.append(line + "\n");
                    }
                    in.close();
                    response = str.toString();
                    Log.i(TAG, "response"+response);
                }
                catch(IllegalStateException exc){

                    exc.printStackTrace();
                }


        } catch(Exception e){

            Log.e("log_tag", "Error in http connection "+response);         

        }
        finally {

        }

        return response;
    }

Java generics - ArrayList initialization

ArrayList<Integer> a = new ArrayList<Number>(); 

Does not work because the fact that Number is a super class of Integer does not mean that List<Number> is a super class of List<Integer>. Generics are removed during compilation and do not exist on runtime, so parent-child relationship of collections cannot be be implemented: the information about element type is simply removed.

ArrayList<? extends Object> a1 = new ArrayList<Object>();
a1.add(3);

I cannot explain why it does not work. It is really strange but it is a fact. Really syntax <? extends Object> is mostly used for return values of methods. Even in this example Object o = a1.get(0) is valid.

ArrayList<?> a = new ArrayList<?>()

This does not work because you cannot instantiate list of unknown type...

When to use "new" and when not to, in C++?

You should use new when you wish an object to remain in existence until you delete it. If you do not use new then the object will be destroyed when it goes out of scope. Some examples of this are:

void foo()
{
  Point p = Point(0,0);
} // p is now destroyed.

for (...)
{
  Point p = Point(0,0);
} // p is destroyed after each loop

Some people will say that the use of new decides whether your object is on the heap or the stack, but that is only true of variables declared within functions.

In the example below the location of 'p' will be where its containing object, Foo, is allocated. I prefer to call this 'in-place' allocation.

class Foo
{

  Point p;
}; // p will be automatically destroyed when foo is.

Allocating (and freeing) objects with the use of new is far more expensive than if they are allocated in-place so its use should be restricted to where necessary.

A second example of when to allocate via new is for arrays. You cannot* change the size of an in-place or stack array at run-time so where you need an array of undetermined size it must be allocated via new.

E.g.

void foo(int size)
{
   Point* pointArray = new Point[size];
   ...
   delete [] pointArray;
}

(*pre-emptive nitpicking - yes, there are extensions that allow variable sized stack allocations).

python inserting variable string as file name

you can do something like

filename = "%s.csv" % name
f = open(filename , 'wb')

or f = open('%s.csv' % name, 'wb')

Retrieving parameters from a URL

The url you are referring is a query type and I see that the request object supports a method called arguments to get the query arguments. You may also want try self.request.get('def') directly to get your value from the object..